Download as pdf or txt
Download as pdf or txt
You are on page 1of 101









                 



 
     







 














 
 


 

 



 

 

 

Page 1 of 99
_________________________________________________________________________________Physics










 
 









 











 











Page 2 of 99
_________________________________________________________________________________Physics


   
           

         

           
      




 

 
       


 
 


    
 
 

 
 
      
  
    

    




   
 
   
      
 
    
 
     
  
 
       
   
     
     
Page 3 of 99
_________________________________________________________________________________Physics

 


    


     

  

 
 

  

  

        

      

        

      


 

      

    

     

   
    

 

 
   

 

         


  

   
           
 

        





  


        
    
 

   

       

 
  
 

     
     
      

Page 4 of 99
_________________________________________________________________________________Physics








 
  
    



     


    



       


 
    

 


 
 


               
 


                 

  
      
     
   


  
 


                    





  
             
  

Page 5 of 99
_________________________________________________________________________________Physics



  
   
 

 
 


   


    
 
 
     
  

         



   
    
   

   




     

 
 



        

  
       
 


  




 
 
           
    






 


  


    
  



Page 6 of 99
_________________________________________________________________________________Physics

PRACTICE PROBLEM

   


                    
             

 

  

 
  

 
 
 
 
 
   
 
 
      
       




 
 
 
 


 

   

 

             

Page 7 of 99
_________________________________________________________________________________Physics

    

  



   



  

      

 
        

      

  





 



    


  
    

 







  

  
    
      
    

 



 
  
      

 

  
    
   
   

Page 8 of 99
_________________________________________________________________________________Physics

    




  
  

 
 








 

  
    

  
 
  
    

     

 


    

 
   

   


 



     

 



Page 9 of 99
_________________________________________________________________________________Physics

 

             


             

  



    
 
 

 
  
 

 


   



   
  
  


     



     
 




  
 
 

 
     
 



 
    
 

 


Page 10 of 99
_________________________________________________________________________________Physics

    


   

   
      
 

 





    


    

  




   


 
    
 

  
     
 
 
 


     

    





 
   
 


   


 

 
 
 


Page 11 of 99
_________________________________________________________________________________Physics

            


              
  
 
 
 






    


      


    
   
   
 
 



 
    

   




  


   
    
 
 




 
  
   
 
 



  


    

   



Page 12 of 99
_________________________________________________________________________________Physics

    


   

  


 
        



         
    

  


  


     
 
 














  



 



 



 
 
 

Page 13 of 99
_________________________________________________________________________________Physics

 

 
   
 

  


 

  
 
 





                
               




  

 
  


        


            




  



  
 
       
     


               

     

Page 14 of 99
_________________________________________________________________________________Physics






 
   
                  
   
 
    
    
      
     
 
      
  

   







  


       
  

 



  
          
 





 
     

 


 


  




      



  
               
  

Page 15 of 99
_________________________________________________________________________________Physics




 

 
 



  
    

     
   






 
    

 

  

 
     
    
       
      






 
          

    







  
      
 

 
  


 

 
 
  




     
                
     

Page 16 of 99
_________________________________________________________________________________Physics








  

  
   


  
     

    
                
  



 

  

     
                
    
 

   
         
   





 


    
    
      
     


 

  




 
     
          
     

Page 17 of 99
_________________________________________________________________________________Physics





  


  
                  
  





  
            
  






 
   
  


    
                 
    



   
 






  
  
  
   
 
    
  
   


   


    

Page 18 of 99
_________________________________________________________________________________Physics

PRACTICE PROBLEM
 

 
 
 

 


 

 
 
 

 

  

 

 

 



  




 


  
  


  

    
    

 
 
 

Page 19 of 99
_________________________________________________________________________________Physics




    


           


 

    
 
  
  

 

 


         
     
 



   
 

 

 
  
    




   
 


 



  

 
  

Page 20 of 99
_________________________________________________________________________________Physics

        


 



 

   
 

  

 
   
        




 
 

          

   


  




     



 

 
 
 





Page 21 of 99
_________________________________________________________________________________Physics

 

  












       



     

     

    

   



 
  

 
 
         

 



 
 

         


   

      





                    
 



                 

Page 22 of 99
_________________________________________________________________________________Physics




 








           

 
  
   
           
 
     



   

 

  





 


  
    
 




 









    
          
  

Page 23 of 99
_________________________________________________________________________________Physics



 



 



  

  

   
           
 
  

 
 
  
   
   

  

PRACTICE PROBLEM

    
      
  

 



 



      

 


   

 

 

    

 
  

 

Page 24 of 99
_________________________________________________________________________________Physics

 


 
 
 
 



   
   
  


     
   
       
   

  





   
              
   
 
   

  





  
              

       

      

 
   

 

 

   

  
 

Page 25 of 99
_________________________________________________________________________________Physics




 
    
 

 
   
 



  


  


   









 

       




 
  
         
      
        
  
        
 
            



        




  
   
 


Page 26 of 99
_________________________________________________________________________________Physics

 


 




   

         








                   

 

                        




    

   
    
 
   




 

        






      
     
  
         
 

 
  

          

Page 27 of 99
_________________________________________________________________________________Physics









 

     

     



     

  

    
       
 
 








   
    
   

 
     

  
        

   

          
 


 


 
       

   
 
     

Page 28 of 99
_________________________________________________________________________________Physics




  

    

 
    
 

    
    

 
     

PRACTICE PROBLEM

 


            

 

 

   
 


     



  


         


      
  

   





           
 




Page 29 of 99
_________________________________________________________________________________Physics




















 




 
  
 


 

 

       


 
  

   
 


   
           
  

   
 

Page 30 of 99
_________________________________________________________________________________Physics

         
        

       

 
   
  



  








  

 
   

       

  



   

   



    
  
 




   



 
        
 
 

   
      

 



  

     
 
   



 

   

 
     
 

Page 31 of 99
_________________________________________________________________________________Physics


 

    

     





     
    


 
 
 

     
       
   
              

   


 
 
 
  

 




    

 
          
 


 


  




 
 
        

   



  


 
 
 



  
         
  

Page 32 of 99
_________________________________________________________________________________Physics


 




 



 

 

        

 
   
  
   
     
  





    
            

    


        



 



  


    

 

 






   
 
 


Page 33 of 99
_________________________________________________________________________________Physics






   
    
   




 
    
      
   

   
 
   


 
                
   


     
   
         

  

     
 




 



                     
                
    


 
    
      
  

 

 
     
   

 
  
  

   
 
   
  
 
 

  


Page 34 of 99
_________________________________________________________________________________Physics





 

     
    
  
  





 
 



                 


   
     
 






       


     



  
   


PRACTICE PROBLEM

   



  
   
   
  

 

 


Page 35 of 99
_________________________________________________________________________________Physics

 
     

 

   

  






 




 

 
            

   

 



           

   
        
   

      
   
        
   
          

  


       
 
     
         
  
   
  
      
 

    

Page 36 of 99
_________________________________________________________________________________Physics



  
    

    
  
    

    
  

           

          


 

   



      
     
    
     
  
   
 

  
         
 
 

        
        
   
  
  

 
  
 
  
 
  
   

   
   

   
  
 
  
 
  
 

         


 
  


         

         

Page 37 of 99
_________________________________________________________________________________Physics

      



  






 
        

     
           
   



   


          

 
  
  


   

   
     
  


   
       
 

  
        

          




  


 
 
 
 


 


          

 
       
   

  
   
 

 
 


 

      

Page 38 of 99
_________________________________________________________________________________Physics

    


   
    
  

      
    

  

  




 


   
          

         
 


 
      
 


  

 

     
   
   


 
    
 

 
   
       

    
 

 
 
           
  
 

 
  
 
     
           

  
        
 
         

Page 39 of 99
_________________________________________________________________________________Physics

       


   
   
   
   
   
    
      
  

   
   
 
 
 

 
 

      


 
 
 



 

 
 
 


 



 


 


         
           

 


 
      
    
   

Page 40 of 99
_________________________________________________________________________________Physics




   



  


   

  
 
         
  

 
   
    
    




   
 
 
    
    
    
   
   
   







                 




    

  
             
  







    


     


   
         
     
  
             

Page 41 of 99
_________________________________________________________________________________Physics



  


   


 

 


 
 



    

   
             
   

   
                                     
   

PRACTICE PROBLEM
 
                
 
         
 
            
       
                    

 

   
 
 

Page 42 of 99
_________________________________________________________________________________Physics

PRACTICE PROBLEM PRACTICE PROBLEM

       

  
   

     

      

   PRACTICE PROBLEM

    

 
PRACTICE PROBLEM


        

 

  
   

 
PRACTICE PROBLEM



  

 
   
  
    

     
 


                

PRACTICE PROBLEM
 
     
   

   
       
     
 
   


Page 43 of 99
Physics
EXERCISE - 1 CLASS WORK
KINEMATICS
1. A body rotating at 20 rad/s is acted upon by a constant torque providing it a deceleration of 2 rad/s2. At what
time will the body have kinetic energy same as the initial value if the torque continues to act?
2. A solid body rotates about a stationary axis according to the law  = at – bt3, where a = 6.0rad/s and b = 2.0
rad/s3. Find:
(a) the mean values of the angular velocity and angular acceleration averaged over the time interval between
t = 0 and the complete stop;
(b) the angular acceleration at the moment when the body stops.
3. A solid body starts rotating about a stationary axis with an angular acceleration  = at , where
 = 2.0 × 10–2 rad/s3. How soon after the beginning of rotation will the total acceleration vector of an arbitrary
point of the body form an angle  = 60º with its velocity vector..
MOMENT OF INERTIA
4. Three particles, each of mass 200 g, are kept at the corners of an equilateral triangle of side 10 cm. Find the
moment of inertia of the system about an axis
(a) joining two of the particles and
(b) passing through one of the particles and perpendicular to the plane of the particles.
5. Particles of masses 1 g, 2 g, 3 g,.........100 g are kept at the marks 1 cm, 2 cm, 3 cm,...... 100 cm respectively
on a metre scale. Find the moment of inertia of the system of particles about a perpendicular bisector of the
metre scale.
6. A uniform triangular plate of mass m whose vertices

are ABC has lengths l,  and  . Find the moment of inertia


2 2
of this plate about an axis passing through point B and perpendicular
to the plane of the plate.
7. Find the moment of inertia of a pair of spheres, each having a mass m and radius r, kept in contact about the
tangent passing through the point of contact.
8. The moment of inertia of a uniform rod of mass m = 0.50 kg and length l = 1 m is I = 0.10 kg -m2 about a line
perpendicular to the rod. Find the distance of this line from the middle point of the rod.
9. Calculate the moment of inertia of a copper uniform disc relative to the symmetry axis perpendicular to the
plane of the disc, if its thickness is equal to b = 2.0 mm and its radius to R = 100 mm;
10. The radius of gyration of a uniform disc about a line perpendicular to the disc equals its radius. Find the distance
of the line from the centre.
11. Find the moment of inertia about x-axis of uniform thin plate of density r kg/m2 as shown in the Figure.
y 2
y=x

x
O a
2
y=–x

Page 44 of 99
Physics
12. Calculate the moment of inertia of a uniform rod of mass m & length l about an axis passing through one end &
making angle θ = 45º with its length.
13. The surface density (mass/area) of a circular disc of radius a depends on the distance from the centre of
r(r) = A + Br. Find its moment of inertia about the line perpendicular to the plane of the disc through its centre.
TORQUE
14. A force F = A î + B ĵ is applied to a point whose radius vector relative to the origin of coordinates O is equal
to r = a î + b ĵ , where a, b &, A, B are constants, and î , ĵ are the unit vectors of the x and y axes. Find the
moment N and the arm  of the force relative to the point O.
  
15. A force F1  A ĵ is applied to a point whose radius vector r1  a î , while a force F2  B î is applied to the point

whose radius vector r2  b ĵ . Both radius vectors are determined relative to the origin of coordinates O, î and
ĵ are the unit vectors of the x and y axes, a, b, A. B are constants. Find the arm  of the resultant force relative
to the point O.
16. A cubical block of mass m and edge a slides down a rough inclined plane of inclination  with a uniform speed.
Find the torque of the normal force acting on the block about its centre.
R OTATIONA L EQU ILIB RIU M

17. A uniform metre scale of mass m is suspended by two vertical string attached

to its two ends as shown in figure. A body of mass m is placed on the 80 cm

mark. Calculate the ratio of tension is string.


18. Assuming frictionless contacts, determine the magnitude of external horizontal force P applied at the lower end
for equilibrium of the rod. The rod is uniform and its mass is 'm'.

19. A uniform ladder of length 10.0 m and mass 16.0 kg is resting against a vertical wall making an angle of 37°
with it. The vertical wall is frictionless but the ground is rough. An electrician weighing 60.0 kg climbs up the
ladder. If he stays on the ladder at a point 8.00 m from the lower end, what will be the normal force and the
force of friction on the ladder by the ground? What should be the minimum of friction for the electrician to work
safely?
20. The door of an almirah is 6 ft high, 1.5 ft wide and weighs 8 kg. The door is supported by two hinges situated
at a distance of 1 ft from the ends. If the magnitudes of the forces exerted by the hinges on the door are equal,
find this magnitude.

Page 45 of 99
Physics
ROTATION ABOUT FIXED AXIS
21. A light rod of length 1 m is pivoted at its centre and two masses of
5 kg and 2 kg are hung from the ends as shown in figure.
(a) Find the initial angular acceleration of the rod assuming that
m2=2 kg m1=5 kg
it was horizontal in the beginning.
(b) If the rod has a mass of 1 kg distributed uniformly over its length.
(i) Find the initial angular acceleration of the rod.
(ii) Find the tension in the supports to the blocks of mass 2 kg and 5 kg.
22. The uniform rod AB of mass m is released from rest when  = 60º. Assuming that the
friction force between end A and the surface is large enough to prevent
sliding, determine (for the instant just after release)
(a) The angular acceleration of the rod
(b) The normal reaction and the friction force at A.
(c) The minimum value of m, compatible with the described motion.
23. A uniform square plate of mass m is supported as shown. If the cable
B
suddenly breaks, determine just after that momentum;
(a) The angular acceleration of the plate.
(b) The acceleration of corner C. A C

(c) The reaction at A. 


24. The pulley shown in fig. has a moment of inertia I about its axis and its radius
I
R
is R. Find the magnitude of the acceleration of the two blocks. Assume
m
that the string is light and does not slip on the pulley ? M

25. The pulleys shown in figure has a radius of 20 cm and moment of inertia 0.2 kg-m2. The
string going over it attached at one end to a vertical spring of spring constant 50 N/m
fixed from below, and supports a 1 kg mass at the other end. The system is released
from rest with the spring at its natural length. Find the speed of the block when it has
descended through 10 cm. Take g = 10 m/s2.
26. A uniform rod pivoted at its upper end hangs vertically. It is displaced through an angle of 60°C and then
released. Find the magnitude of the force acting on a particle of mass dm at the tip of the rod when the rod
makes an angle of 37° with the vertical.
A N G ULA R MO MEN TU M
27. A particle having mass 2 kg is moving along straight line 3x + 4y = 5 with speed 8m/s. Find angular momentum
of the particle about origin. x and y are in meters.
28. A uniform square plate of mass 2.0 kg and edge 10 cm rotates about one of its diagonals under the action of a
constant torque of 0.10 N.m. Calculate the angular momentum and the kinetic energy of the plate at the end of
the fifth second after the start.

Page 46 of 99
Physics
29. A diver having a moment of inertia of 6.0 kg-m about an axis through its centre of mass rotates at an angular
2

speed of 2 rad/s about this axis. If he folds his hands and feet to decrease the moment of inertia to 5.0 kg-m2,
what will be the new angular speed ?
30. A cylinder of mass 5 kg and radius 30 cm, and free to rotate about its axis, receives an angular impulse of
3 kg m2 s–1 initially followed by a similar impulse after every 4s. What is the angular speed of the cylinder after
30s of the initial impulse ? The cylinder is at rest initially.
COMBINED TRANSLATIONAL + ROTATION MOTION
31. The end B of rod AB which makes angle q with the floor is being pulled with a velocity v0 as shown. Taking the
length of the rod as l, calculate the following at the instant when θ = 37º
(a) The velocity of end A
(b) The angular velocity of rod
(c) Velocity of CM of the rod.

32. A point A is located on the rim of a wheel of radius R = 0.50 m which rolls without slipping along a horizontal
surface with velocity v = 1.00 m/s. Find:
(a) the modulus and the direction of the acceleration vector of the point A ;
(b) the total distance s traversed by the point A between the two successive moments at which it touches the
surface.

33. A tangential force F acts at the top of a thin spherical shell of mass m and F

R
radius R. Find the acceleration of the shell if it rolls without slipping.
f
34. A ball of radius R =10.0 cm rolls without slipping down an inclined plane so that its centre moves with constant
acceleration w = 2.50 cm/s2; t = 2.00 s after the beginning of motion its position corresponds to that shown in
Fig. Find :
(a) the velocities of the points A, B and O ;
(b) the accelerations of these points.
35. A cylinder rolls without slipping over a horizontal plane. The radius of the cylinder is equal to r. Find the
curvature radii of trajectories traced out by the points A and B in
Fig

36. Determine the point of the rod AB of length 0.9 m at which the force P should be
applied if the acceleration of point B is to be zero. Knowing that the magnitude of P is
2.70 N, determine the corresponding angular acceleration of the rod and the acceleration
of the center of the rod. ( Mass of the rod is 1.5 kg. Force P is
to be applied perpendicular to the rod and the rod is in horizontal smooth plane).

Page 47 of 99
Physics
37. A uniform rod 1.0m long is initially standing vertically on a smooth horizontal surface. It is struck a sharp
horizontal blow at the top end, with the blow directed at right angles to the rod axis. As a result, the rod
acquired an angular velocity of 3 rad/sec.
(a) What is the translational velocity of the centre of mass of the rod after the blow.
(b) Which point on the rod is stationary on the rod just after the blow.
38. A small spherical ball is released from a point at a height h on a rough track shown in figure. Assuming that it
does not slip anywhere, find its linear speed when it rolls on the horizontal part of the track.

39. A hollow sphere is released from the top of an inclined plane of inclination θ .
(a) What should be the minimum coefficient of friction between the sphere and the plane to prevent sliding.
( b ) F i n d t h e k i n e t i c l on the incline if the friction coefficient is half
e n e r g y o f t h e b a l l a s i t m o v e s d o w n a l e n g t h

the value calculated in part (a).


40. A solid sphere of mass 0.50 kg is kept on a horizontal surface.The coefficient of static friction between
the surfaces in contact is 2/7. What maximum force can be applied at the highest point in the horizontal
direction so that the sphere does not slip on the surface?
41. Figure shows a rough track, a portion of which is in from of a cylinder of radius R. With what minimum linear
speed should a sphere of radius r be set rolling on the horizontal part so that it completely goes round the circle
on the cylindrical part.

42. In the arrangement shown in the figure weight A possesses mass m, a pulley B possesses mass M. Also known
are the moment of inertia I of the pulley relative to its axis and the radii of the pulley are R and 2R respectively.
Consider the mass of the threads is negligible. Find the acceleration of weight A after the system is set free.
(Assume no slipping takes place anywhere)

B
B
2R

R A

Page 48 of 99
Physics
43. Two small spheres A & B respectively of mass m & 2m are connected by a rigid rod of length l & negligible
mass. The two spheres are resting on a horizontal, frictionless surface. When A is suddenly given the velocity v0
as shown. Find velocities of A & B after the rod has rotated through 180º.

A v0

44. A ring rolls on a horizontal surface without sliding. The velocity of the centre is v. It encounters a step of height
0.3 R where R is the radius of the ring. Calculate the angular velocity of the ring just after the impact. Assume
that the ring does not return back. (and there is sufficient friction to avoid slipping) . Find the minimum value of
' v' so that the ring ascends the step ..
45. A uniform wheel of radius R is set into rotation about its axis at an angular speed  . This rotating wheel is now
placed on a rough horizontal surface with its axis horizontal. Because of friction at the contact, the wheel
accelerates forward and its rotation decelerates till the wheel starts pure rolling on the surface. Find the linear
speed of the wheel after it starts pure rolling.
46. A thin spherical shell lying on a rough horizontal surface is hit by a cue in such a way that the line of action passes
through the centre of the shell. As a result, the shell starts moving with a linear speed v without any initial angular
velocity. Find the linear speed of the shell after it starts pure rolling on the surface.
47. A sphere of mass m and radius r is projected along a rough horizontal surface with the initial velocities indicated.
If the final velocity of the sphere is to be zero, express ;
(a) The required w0 in terms of  0 and r..
(b) Thetime requiredfor the sphere to come torest in terms of  0 &
the coefficient of friction  k .

Page 49 of 99
Physics
EXERCISE - 2 BAISC OBJECTIVE QUESTIONS
KINEMATICS
1. A chain couples and rotates two wheels in a bicycle. The radii of bigger and smaller wheels in a bicycle.
The radii of bigger and smaller wheels are 0.5m and 0.1. respectively. The bigger wheel rotates at the
rate of 200 rotations per minute, then the rate of rotation of smaller wheel will be -
(A) 1000 rpm (B) 50/3 rpm (C) 200 rmp (D) 40 rpm

2. If the position vector of a particle is r = (3 i + 4 j ) metre and its angular velocity is  = ( j + 2 k )
rad/sec then its linear velocity is (in m/s)

(A) –(8 i –6 j + 3 k ) (B) (3 i + 6 j + 8 k )

(C) –(3 i + 6 j + 6 k ) (D) (6 i + 8 j + 3 k )


3. A car is moving with a speed of 72 Km/hour. The diameter of its wheels is 50cm. If its wheels come
to rest after 20 rotations as a result of application of brakes, then the angular retardation produced in
the car will be
(A) 25.5 Radians/sec2 (B) 0.25 Radians/sec2 (C) 2.55 Radians/sec2 (D) 0
4. A particle, situated in an object, moves with angular acceleration of 6 rad/sec 2 and with 2 2 rad/sec
angular velocity. If the radius of the circular path is 1m, its total acceleration in m/sec 2 will be :
(A) 1 (B) 100 (C) 10 (D) 10
5. Two particles of masses m1 and m2 complete one revolution of respective radii r1 and r2 in same time.
The ratio of their angular speeds would be -
(A) m1r12 : m2r22 (B) r1 : r2 (C) r2 : r1 (D) 1 : 1
6. A particle, moving along a circular path has equal magnitudes of linear and angular acceleration. The
diameter of the path is (in metre) :
(A) 1 (B) π (C) 2 (D) 2 π
MOMENT OF INERTIA
7. The moment of inertia of a body depends upon -
(A) mass only (B) angular velocity only
(C) distribution of particles only (D) mass and distribution of mass about the axis
8. On account of melting of ice at the north pole the moment of inertia of spinning earth -
(A) increases (B) decreases
(C) remains unchanged (D) depends on the time
9. Two spheres of same mass and radius are in contact with each other. If the moment of inertia of a sphere
about its diameter is I, then the moment of inertia of both the spheres about the tangent at their common
point would be -

(A) 3I (B) 7I (C) 4I (D) 5I


10. The moment of inertia of a sphere of radius R about an axis passing through its centre is proportional to -
(A) R2 (B) R3 (C) R4 (D) R5

Page 50 of 99
Physics
11. The moment of inertia of NaCl molecule with bond length r about an axis perpendicular to the bond and
passing through the centre of mass is
mNa  m Cl mNa x m Cl mNa  m Cl
(A) (mNa + mCl)r2 (B) m xm r2 (C) m r2 (D) m  m r2
Na Cl Na  m Cl Na Cl

12. A disc of metal is melted to recast in the form of a solid sphere. The moment of inertia about a vertical
axis passing through the centre would -
(A) decrease (B) increase (C) remains same (D) nothing can be said
13. A solid sphere and a hollow sphere of the same mass have the same moments of inertia about their
respective diameters, the ratio of their radii is
(A) (5)1/2 : (3)1/2 (B) (3)1/2 : (5)1/2 (C) 3 : 2 (D) 2 : 3
14. A stone of mass 4kg is whirled in a horizontal circle of radius 1m and makes 2 rev/sec. The moment
of inertia of the stone about the axis of rotation is
(A) 64 kg × m2 (B) 4 kg × m2 (C) 16 kg × m2 (D) 1 kg × m2
15. In an arrangement four particles, each of mass 2 gram are situated at the coordinate points (3, 2, 0),
(1, –1, 0), (0, 0, 0) and (–1, 1, 0). The moment of inertia of this arrangement about the Z-axis will be
(A) 8 units (B) 16 units (C) 43 units (D) 34 units
16. Two discs have same mass and thickness. Their materials are of densities r 1 and r2. The ratio of their
moment of inertia about central axis will be -
(A) r1 : r2 (B) r1r2 : 1 (C) 1 : r1r2 (D) r2 : r1
17. Three rings, each of mass P and radius Q are arranged as shown in the figure. The moment of inertia
of the arrangement about YY’ axis will be
7
(A) PQ2
2
2
(B) PQ2
7

2
(C) PQ2
5
5
(D) PQ2
2

18. Three thin uniform rods each of mass M and length L and placed along the three axis of a Cartesian
coordinate system with one end of each rod at the origin. The M.I. of the system about z-axis is

ML2 2ML2 ML2


(A) (B) (C) (D) ML2
3 3 6
19. A circular disc A of radius r is made from an iron plate of thickness t and another circular disc B of radius
4r is made from an iron plate of thickness t/4. The relation between the moments of inertia I A and IB
is
(A) IA > IB (B) IA = IB
(C) IA < IB (D) depends on the actual values of t and r.

Page 51 of 99
Physics
TORQUE
20. The torque needed to produce an angular acceleration of 18rad/sec 2 in a body of moment of inertia
2.5kg-m2 would be -
(A) 4.5 newton - metre (B) 45 newton-metre
(C) 4.5 × 102 newton-metre (D) 45 × 10–2 newton- metre
21. A wheel starting with angular velocity of 10 radian/sec acquires angular velocity of 100 radian/sec in 15
seconds. If moment of inertia is 10kg-m2, then applied torque (in newton-metre) is
(A) 900 (B) 100 (C) 90 (D) 60
22. The moment of inertia and rotational kinetic energy of a fly wheel are 20kg-m2 and 1000 joule respectively.
Its angular frequency per minute would be -
600 25 5 300
(A) (B) (C) (D)
 2  
23. A rigid body is rotating about a vertical axis at n rotations per minute, If the axis slowly becomes
horizontal in t seconds and the body keeps on rotating at n rotations per minute then the torque acting
on the body will be, if the moment of inertia of the body about axis of rotation is I.
2nI 2 2 nI 4nI
(A) zero (B) (C) (D)
60t 60t 60t
24.      
A force of (2 i – 4 + 2 k ) Newton acts at a point (3 i + 2 – 4 k ) metre from the origin. The
j j
magnitude of torque is
(A) zero (B) 24.4 N-m (C) 0.244 N-m (D) 2.444 N-m
The angular velocity of a body is  = 2 i + 3 j + 4 k and a torque  = i + 2 j + 3 k acts on
 
25.
it. The rotational power will be
(A) 20 watt (B) 15 watt (C) 17 watt (D) 14 watt
26. A torque of 2 newton-m produces an angular acceleration of 2 rad/sec2 a body. If its radius of gyration
is 2m, its mass will be :
(A) 2kg (B) 4 kg (C) 1/2 kg (D) 1/4 kg
27. The three similar torque (t) are acting at an angle of 120º with each other. The resultant torque will be:
(A) zero (B) 2 t (C) 3t (D) t/3
28. A ring of diameter 1m is rotating with an angular momentum of 10 Joules-sec. The necessary tangential
force required to increase its angular momentum by 50% in 1sec will be (in newtons) :
(A) 10 (B) 5 (C) 15 (D) 20
29. A particle is at a distance r from the axis of rotation. A given torque t produces some angular acceleration
in it. If the mass of the particle is doubled and its distance from the axis is halved, the value of torque
to produce the same angular acceleration is
(A) t/2 (B) t (C) 2t (D) 4t

Page 52 of 99
Physics
ROTATIONAL EQUILIBRIUM
30. A weightless rod is acted on by upward parallel forces of 2N and 4N at ends A and B respectively. The total length
of the rod is AB =3 m. To keep the rod in equilibrium a force of 6N should act in the following manner:–
(A) Downwards at any point between A and B (B) Downwards at mid point of AB
(C) Downwards at a point C such that AC =1m (D) Downwards at a point D such that BD =1m
31. In an experiment with a beam balance an unknown mass m is balanced by two known masses of 16kg and
4 kg as shown in figure. The value of the unknown mass m is :–

1 2 1 2

16kg m m 4kg

(A) 10 kg (B) 6 kg (C) 8 kg (D) 12 kg


32. In figure the uniform gate weighs 300 N and is 3 m wide and 2 m high. It is supported cable
3m
by a hinge at the bottom left corner and a horizontal cable at the top left
corner, as shown. Find the tension in the cable and the force that the 2m
hinge exerts on the gate.
(A) 225 N, 225 N, 300 N (B) 450 N, 450 N, 300 N
hinge
(C) 225 N, 300 N, 225 N (D) 450 N, 300 N, 450 N
33. A uniform rod of length L rests against a smooth wall as shown in figure. Find the
friction coefficient between the ground and the lower end if the minimum angle
that the rod can make with the horizontal is θ .( θ = 37o , L = 125 m, h = 45m.)
(A) 5/7
(B) 6/7
(C) 2/7
(D) 1/7
34. Figure shows a vertical force F that is applied tangentially to a uniform cylinder of weight W. The coefficient of
static friction between the cylinder and all surfaces is 0.5. Find in terms of W, the maximum force that can be
applied without causing the cylinder
to rotate.

(A) 3W/8 (B) 2W/5 (C) 5W/8 (D) 2W/3


35. A body is in equilibrium under the influence of a number of forces. Each force has a different line of action.
The minimum number of forces required is
(A) 2, if their lines of action pass through the centre of mass of the body
(B) 3, if their lines of action are not parallel
(C) 3, if their lines of action are parallel
(D) 4, if their lines of action are parallel and all the forces have the same magnitude

Page 53 of 99
Physics
ROTATION ABOUT FIXED AXIS
36. A string is wrapped around the rim of a wheel of moment of inertia 0.20 kg-m2 and radius 20 cm. The wheel
is free to rotate about its axis and initially the wheel is rest. The string is now pulled by a force of 20N. The
angular velocity of the string after 5 seconds will be :–
20N
(A) 90 rad/s
(B) 70 rad/s
(C) 95 rad/s
(D) 100 rad/s
37. In the following figure r1 and r2 are 5 cm and 30 cm respectively. If the moment of inertia of the wheel is 5100
kg-m2 then its angular acceleration will be :- 10N

(A) 10–4 rad/sec2 (B) 10–3 rad/sec2 30


°
r1

(C) 10 rad/sec
–2 2
(D) 10 rad/sec
–1 2
12N
9N
r2
38. A string is wrapped on a wheel of moment of inertia 0.20 kg -m2 and radius 10cm
and goes through a light pulley to support a block of mass 2.0 kg as shown in
figure. 0.20 kg -m2 10 cm 2.0 kg
(a) If the smaller pulley is assume to be light, find the acceleration of the block.
(A) g/11 (B) 2g/11 (C) g/13 (D) 2g/13
(b) The smaller pulley has its radius 5.0 cm and moment of inertia 0.10 kg-m2.
Find the tension in the part of the string joining the pulleys.
(A) 10g/31 (B) 5g /31 (C) 4g/31 (D) 20g/31

39. A solid cylinder of mass M = 1kg & radius R = 0.5m is pivoted at its centre & has
three particles of mass m = 0.1kg mounted at its perimeter as shown in
the figure. The system is originally at rest. Find the angular speed
of the cylinder, when it has swung through 900 in anticlockwise direction.
(A) 5 rad/s (B) 10 rad/s

(C) 7 rad/s (D) 20 rad/s


ANGULAR MOMENTUM
40. The torque applied to a ring revolving about its own axis so as to change its angular momentum by 2
J-s. in 5 s, is
(A) 10N-m (B) 2.5 N-m (C) 0.1N-m (D) 0.4N-m
41. The angular velocity of a body changes from ω 1 to ω 2 without applying torque. The ratio of initial radius of
gyration to the final radius of gyration is :–
(A) 2 : 1 (B) 1 : 2 (C) ω 2: ω 1 (D) ω 1 : ω 2
42. A dog of mass m is walking on a pivoted disc of radius R and mass M in a circle of radius R/2 with
an angular frequency n: the disc will revolve in opposite direction with frequency -
mn mn 2mn 2Mn
(A) (B) (C) (D)
M 2M M m

Page 54 of 99
Physics
ROTATIONAL KINETIC ENERGY
43. A circular ring of wire of mass M and radius R is making n revolutions/sec about an axis passing through
a point on its rim and perpendicular to its plane. The kinetic energy of rotation of the ring is given by-
1
(A) 4  2MR2n2 (B) 2  2MR2n2 (C)  2MR2n2 (D) 8  2MR2n2
2
44. Rotational kinetic energy of a given body about an axis is proportional to -
(A) time period (B) (time period)2 (C) (time period)-1 (D) (time period)–2
45. A circular disc has a mass of 1kg and radius 40 cm. It is rotating about an axis passing through its centre
and perpendicular to its plane with a speed of 10rev/s. The work done in joules in stopping it would be-
(A) 4 (B) 47.5 (C) 79 (D) 158
46. A thin bar of length L is suspended from one end and rotated at a speed of n revolutions per second.
The rotational kinetic energy of the bar is -
(A) 2ML2  2n2 (B) 1/2 ML2  2n2. (C) 2/3 ML2  2n2 (D) 1/6 ML2  2n2.
ROTATION + TRANSLATION MOTION
47. A cylinder of mass M and radius R rolls on an inclined plane. The gain in kinetic energy is
1 1 3 3
(A) Mv2 (B) I 2 (C) Mv2 (D) I 2
2 2 4 4
48. The condition that a rigid body is rolling without slipping on an inclined plane is
(A) it has acceleration less than g.
(B) it has rotational and translational K.E. to be equal
(C) it has linear velocity equal to radius times angular velocity
(D) the plane is fricitionless.
49. A disk and a ring of the same mass are rolling to have the same kinetic energy. What is ratio of their
velocities of centre of mass
(A) (4:3)1/2 (B) (3 : 4)1/2 (C) (2)1/2 : (3) 1/2 (D) (3)1/2 : (2)1/2
50. The acceleration down the plane of spherical body of mass m radius R and moment of inertia I having
inclination q to the horizontal is

g sin  g sin  g sin  g sin


(A) 2 2 (B) 2 (C) 2 (D)
1 I / R 1 I / R 1  I / MR MR2  I

51. A ring takes time t1 in slipping down an inclined plane of length L, whereas it takes time t2 in rolling
down the same plane. The ratio of t 1 and t2 is -
(A) (2)1/2 : 1 (B) 1 : (2)1/2 (C) 1 : 2 (D) 1 : 21/4
52. A solid cylinder starts rolling from a height h on an inclined plane. At some instant t, the ratio of its
rotational K.E. and the total K.E. would be
(A) 1 : 2 (B) 1 : 3 (C) 2 : 3 (D) 1 : 1
53. When different regular bodies roll down along an inclined plane from rest, then acceleration will be
maximum for a body whose -
(A) radius of gyration is least (B) mass is least
(C) surface area is maximum (D) moment of inertia is maximum

Page 55 of 99
Physics
54. If a ring, a disc, a solid sphere and a cylinder of same radius rolls down on inclined plane, the first one to reach
the bottom will be :–
(A) disc (B) ring (C) solid sphere (D) cylinder
55. A solid cylinder of mass M and radius R rolls without slipping down an inclined plane of length L and height
h. What is the speed of its centre of mass when the cylinder reaches its bottom :–
3 4
(A) 2 gh (B) gh (C) gh (D) 4 gh
4 3
56. If the applied torque is directly proportional to the angular displacement θ , then the work done in rotating
the body through an angle θ would be - (C is constant of proportionality)
1 1
(A) C θ (B) Cθ (C) Cθ2 (D) C θ 2
2 2
57. On account of the earth rotating about its axis :-
(A) the linear velocity of objects at equator is greater than at other places
(B) the angular velocity of objects at equator is more than that of objects at poles
(C) the linear velocity of objects at all places at the earth is equal, but angular velocity is different
(D) at all places the angular velocity and linear velocity are uniform
A
58. In the adjoining figure along which axis the moment of inertia of
the triangular lamina will be maximum- [Given that AB < BC < AC]
(A) AB
(B) BC
C
(C) CA B
(D) For all axis
59. A circular disc is to be made by using iron and aluminium so that it acquired maximum moment of inertia about
geometrical axis. It is possible with :–
(A) aluminium at interior and iron surrounded to it.
(B) iron at interior and aluminium surrounded to it.
(C) using iron and aluminium layers in alternate order.
(D) sheet of iron is used at both external surface and aluminium sheet as internal layer.
E
60. We have a rectangular slab of same thickness. E, F, G, H are the middle point A B
of AB, BC, CD and AD respectively then which of the following axis the moment of in-
ertia will be minimum :– H F

(A) AD (B) EG
D C
(C) BD (D) HF G
61. Four similar point masses (each of mass m) are placed on the circumference of a disc of mass M and radius
R. The M.I. of the system about the normal axis through the centre O will be:-
1
(A) MR2 + 4mR2 (B) MR2 + 4mR2
2 O
8
(C) MR2 + mR2 (D) None of these
5

Page 56 of 99
Physics
62. Two rings of the same radius and mass are placed such that their centres are at a common point and their planes
are perpendicular to each other. The moment of inertia of the system about an axis passing through the centre
and perpendicular to the plane of one of the rings is (mass of the ring = m, radius = r) :–
1 3
(A) mr2 (B) mr2 (C) mr2 (D) 2mr2
2 2
63. Three point masses, each of m, are placed at the corners of an equilateral triangle of side l. Then the moment
of inertia of this system about an axis along one side of the triangle is :–
3 3
(A) 3 ml2 (B) ml2 (C) ml2 (D) ml2
4 2
64. Two rods each of mass m and length l are joined at the centre to form a cross. The moment of inertia of this cross
about an axis passing through the common centre of the rods and perpendicular to the plane formed by them, is :–
m 2 m 2 m 2 m 2
(A) (B) (C) (D)
12 6 3 2
65. If a body completes one revolution in p sec then the moment of inertia would be:–
(A) Equal to rotational kinetic energy (B) Double of rotational kinetic energy
(C) Half of rotational kinetic energy (D) Four times of the rotational kinetic energy
66. For the same total mass which of the following will have the largest moment of inertia about an axis passing
through its centre of mass and perpendicular to the plane of the body
(A) a disc of radius a (B) a ring of radius a
(C) a square lamina of side 2a (D) four rods forming a square of side 2a
67. The axis X and Z in the plane of a disc are mutually perpendicular and Y-axis is perpendicular to the plane
of the disc. If the moment of inertia of the body about X and Y axes is respectively 30 kg m2 and 40 kgm2
then M.I. about Z-axis in kg m2 will be:–
(A) 70 (B) 50 (C) 10 (D) Zero
68. A person supports a book between finger and thumb as shown (the point of grip is
assumed to be at the corner of the book). If the book has a weight of
W then the person is producing a torque on the book of a
a b
(A) W anticlockwise (B) W anticlockwise
2 2 b
(C) Wa anticlockwise (D) Wa clockwise
69. If the earth is a point mass of 6 × 10 kg revolving around the sun at a distance of 1.5 × 108 km and in
24

time T= 3.14 × 107 second, then the angular momentum of the earth around the sun is :–
(A) 1.2 × 1018 kg m2/s (B) 1.8 × 1020 kg m2/s (C) 1.5 × 1037 kg m2/s (D) 2.7 × 1040 kg m2/s
70. A circular turn table has a block of ice placed at its centre. The system rotates with an angular speed  about
an axis passing through the centre of the table. If the ice melts on its own without any evaporation, the speed
of rotation of the system :–
(A) becomes zero (B) remains constant at the same value of 
(C) increases to value greater than  (D) decreases to a value less than 

Page 57 of 99
Physics
71. A boy stands over the centre of a horizontal platform which is rotating freely with a speed of
2 revolutions/s about a vertical axis through the centre of the platform and straight up through the boy. He holds
2 kg masses in each of his hands close to his body. The combined moment of inertia of the system is 1 kg-
m.2. The boy now stretches his arms so as to hold the masses far from his body. In this situation the moment
of inertia of the system increases to 2 kg-m.2. The kinetic energy of the system in the latter case as compared
with that in the previous case will-
(A) Remain unchanged (B) Decrease (C) Increase (D) Remain uncertain
72. A rigid body of mass m rotates with angular velocity w about an axis at a distance d from the centre of mass G. The
radius of gyration about a parallel axis through G is K. The kinetic energy of rotation of the body is :–
1 1 1 1
(A) mk 2 2 (B) md 2 2 (C) m(d 2  k 2 )2 (D) m (d  k ) 2  2
2 2 2 2

Page 58 of 99
Physics
EXERCISE - 3 MAINS LEVEL EXERCISE
SECTION - A OBJECTIVE TYPE

X m C
1. Three particles, each of mass m are situated at the vertices of an equilateral
triangle ABC of side l cm (as shown in the figure). The moment of inertia
of the system about a line AX perpendicular to AB and in the plane of m
ABC, in gram cm2 units will be :– m
A B
Y

5 3 3
(A) 2 ml2 (B) ml2 (C) ml2 (D) ml2
4 2 4

1
2. The moment of inertia of a rod about an axis through its centre and perpendicular to it is ML2 (where M
12
is the mass and L is the length of the rod). The rod is bent in the middle so that the two half make an angle
of 60°. The moment of inertia of the bent rod about the same axis would be :–

1 1 1 ML3
(A) ML2 (B) ML2 (C) ML2 (D)
48 12 24 8 3

3. Two rods of equal mass m and length l lie along the x axis and y axis with their centres origin. What is the
moment of inertia of both about the line x=y :
m 2 m 2 m 2 m 2
(A) (B) (C) (D)
3 4 12 6
4. A rigid body can be hinged about any point on the x-axis. when it is hinged such that the hinge is at x, the moment
of inertia is given by I = x2–2x + 99. The x-coordinate of centre of mass is :–
(A) x=2 (B) x=0 (C) x=1 (D) x=3

5. A rod of mass M and length L is placed in a horizontal plane with one end hinged about the vertical axis. A
Mg 5L
horizontal force of F= is applied at a distance from the hinged end. The angular acceleration of the
2 6
rod will be :-
4g 5g 3g 4g
(A) (B) (C) (D)
5L 4L 4L 3L
6. In the figure (A) half of the meter scale is made of wood while the other half of steel. The wooden part is pivoted
at O. A force F is applied at the end of steel part. In figure (B) the steel part is pivoted at O' and the same
force is applied at the wooden end:–
wood steel steel wood
(A) More angular acceleration will be produced in (A)
O P O' P'
(B) More angular acceleration will be produced in (B)
(A) F (B) F
(C) Same angular acceleration will be produced in both conditions
(D) Information is incomplete

Page 59 of 99
Physics
7. A non uniform rod OA of liner mass density    0 x  0  co nst.
is suspended from ceiling with hinge joint O & light string as
shown in figure. Find the angular acceleration of rod just after O g
the string is cut x
A
2g g
(A) (B)
L L

4g
(C) (D) None of these
3L
8. A particle of mass m moves with a constant velocity. Which of the following statements is not correct about
its angular momentum : Y
D E
(A) it is zero when it is at A and moving along OA
A C
(B) the same at all points along the line DE
(C) of the same magnitude but oppositely directed at B and D B
X
O
(D) increases as it moves along the line BC
9. A thin rod of mass M and length L is struck at one end by a ball of clay
of mass m, moving with speed v as shown in figure. The ball sticks to the rod.
900
After the collision, the angular momentum of the clay-rod system about A, the
midpoint of the rod, is A

 M   vL   M   vL  mvL
(A)  m  3   2  (B)  m  12   2  (C) (D) mvL
2

1
10. If the earth were to suddenly contract to th of its present radius without any change in its mass then the duration
n
of the new day will be nearly :–
24 24
(A) hour (B) 24n hour (C) hour (D) 24n2 hour
n n2

11. A thin circular ring of mass M and radius ‘r’ is rotating about its axis with a constant angular velocity  . Four
objects each of mass m, are kept gently to the opposite ends of two perpendicular diameters of the ring. The
new angular velocity of the ring will be :–

M M (M  4m ) (M  4m )
(A) (B) M  4m (C) (D) M  4m
4m M

12. A horizontal platform is rotating with uniform angular velocity around the vertical axis passing through its centre.
At some instant of time a viscous fluid of mass ‘‘m’’ is dropped at the centre and is allowed to spread out
and finally fall. The angular velocity during this period :–
(A) Decreases continuously (B) Decreases initially and increases again
(C) Remains unaltered (D) Increases continuously

Page 60 of 99
Physics
13. A particle starts from the point (0m, 8m) and moves with uniform velocity of 3ˆi m/s . After 5 seconds, the
angular velocity of the particle about the origin will be y

3m/s
8 3
(A) rad/s (B) rad/s
289 8
8m
24 8
(C) rad/s (D) rad/s x
289 17 O

14. A rod is hinged at its centre and rotated by applying a constant torque starting from rest. The power developed
by the external torque as a function of time is :–

Pext Pext Pext Pext

(A) (B) (C) (D)

time time time time

15. A disc of mass M and radius R rolls on a horizontal surface and then rolls up an inclined plane as shown in
the figure. If the velocity of the disc is v, the height to which the disc will rise will be :–
3v2
(A) 3v (B) 4 g
2

2g
h
v2 v2 v
(C) 4 g (D) 2 g

16. A solid sphere is rolling on a frictionless surface, shown in figure with a translational velocity v m/s. If it is to
climb the inclined surface then v should be:–

v h

(A) ³ 10 7 gh (B) > 2 gh (C) 2gh (D) 10/7gh


17. A rod hinged at one end is released from the horizontal position as shown C A
in the figure. When it becomes vertical its lower half separates without
exerting any reaction at the breaking point. Then the maximum angle '  ' 
made by the hinged upper half with the vertical is B
(A) 300 (B) 450
(C) 600 (D) 900 C
18. There is rod of length l. The velocities of its two ends are v1 and v2 in opposite directions normal to the
rod. The distance of the instantaneous axis of rotation from v1 is :–
v2 v1
(A) Zero (B) v  v  (C) v  v (D) 2l
1 2 1 2

Page 61 of 99
Physics
SECTION - B SUBJECTIVE TYPE
1. Find out the moment of inertia of the following structure (written as PHYSICS) about axis AB and made
of thin uniform rods of the mass per unit length l.
     

A B

2. Calculate the moment of inertia of a wheel about its axis which having rim of mass 24M and twenty four spokes
each of mass M and length l.
3. A carpet of mass M made of inextensible material is rolled along its length in the form of a cylinder of radius
R and is kept on a rough floor. The carpet starts unrolling without sliding on the floor when a negligibly small
push is given to it. Calculate the horizontal velocity of the axis of the cylindrical part of the carpet when
R
its radius reduces to .
2

4. A homogeneous rod AB of length L = 1.8 m and mass M is pivoted at S


the centre O in such a way that it can rotate freely in the vertical plane v
(fig.). The rod is initially in the horizontal position. An insect S of the
same mass M falls vertically with speed v on the point C, midway between
the points O and B. Immediately after falling, the insect moves towards A O C B
the end B such that the rod rotates with a constant angular velocity  . L/2 L/4 L/4

(i) Determine the angular velocity w in terms of v and L.


(ii) If the insect reaches the end B when the rod has turned through an angle of 90°, determine v.

5. Two uniform rods A and B of length 0.6 m each and of masses 0.01 kg P

and 0.02 kg respectively are rigidly joined end to end. The combination is A
pivoted at the lighter end, P as shown in figure. Such that it can freely rotate
about point P in a vertical plane. A small object of mass 0.05 kg, moving
orizontally, hits the lower end of the combination and sticks to it. What should B

be the velocity of the object, so that the system could just be raised to the
horizontal position.

6. Determine the minimum co-efficient of friction between a thin rod and a floor at which a person can slowly
lift the rod from the floor without slipping, to the vertical position applying to its end a force always perpendicular
to its length.

Page 62 of 99
Physics
Y
7. A block X of mass 0.5 kg is held by a long massless string on a frictionless

inclined plane of inclination 30° to the horizontal. The string is wound on a


X
uniform solid cylindrical drum Y of mass 2 kg and of radius 0.2 m as shown
in figure. The drum is given an initial angular velocity such that the block X 30°

starts moving up the plane.

(i) Find the tension in the string during the motion.

(ii) At a certain instant of time the magnitude of the angular velocity of Y is 10 rad s–1.

Calculate the distance travelled by X from that instant of time until it comes to rest.
8. Two thin circular disc of mass 2 kg and radius 10 cm each are joined
by a rigid massless rod of length 20 cm. The axis of the rod is along
the perpendicular to the planes of the disc through their centres. This O
object is kept on a truck in such a way that the axis of the object is
horizontal and perpendicular to the direction of motion of the truck. Its 20cm
friction with the floor of the truck is large enough, so that the object
can roll on the truck without slipping.
Take x-axis as the direction of motion of the truck and z-axis as the vertically upwards direction. If the truck
has an acceleration 9 m/s2, calculate :
(i) the force of friction on each disc and
(ii) the magnitude and direction of the frictional torque acting on each disc about the centre of mass O of
the object. Express the torque in the vector form in terms of unit vector î , ĵ and k̂ in x, y and z-directions.

1
9. Four 2kg masses are connected by m long spokes to an axle as in shown
4

1
figure. A force F of 24N acts on a lever m long to produce an angular
2

acceleration a. Determine the magnitude of a.

10. A uniform rod of mass 8m and length 6a is lying on a horizontal table.

Two point masses m and 2m moving with speed 2v and v respectively


strike the rod and stick to it as shown in figure then-
(i) Calculate the speed of centre of mass of rod after the collision.
(ii) Calculate angular velocity of the rod about an axis passing through its centre of mass.
(iii) Kinetic energy of system after collision.

Page 63 of 99
Physics
11. A stick of length L and mass M lies on a frictionless horizontal surface on which

it is free to move in anyway. A ball of mass m moving with speed v as shown

in fig. What must be the mass of the ball so that it remains at rest immediately

after collision.

12. A rod of length l and mass M held vertically is let go down, without slipping at the point of contact. What is

the velocity of the top end at the time of touching the ground ?

13. An initial momentum is imparted to a homogeneous cylinder as a result of which it begins to roll without slipping

up an inclined plane at speed v0 = 4ms–1. The plane makes an angle of 30° with the horizontal. What time

does the cylinder take before stopping.

14. As shown in the figure, a rod moves with v=2 m/sec and rotates with   2  rad/sec.
2m/s

Find the point on the rod whose velocity is zero in this frame.

Page 64 of 99
Physics
EXERCISE - 4 ADVANCED LEVEL EXERCISE
SECTION - A SINGLE CORRECT QUESTIONS
O
1. A thin rod of length 4l, mass 4m is bent at the points as shown in the fig.
What is the moment of inertia of the rod about the axis passing point 90° 90°

O & perpendicular to the plane of the paper


m 2 10m 2 m 2 m 2
(A) (B) (C) (D)
3 3 12 24

2. A smooth tube of certain mass is rotated in gravity free space and released.
The two balls shown in the figure move towards ends of the tube. For the
whole system which of the following quantity is not conserved :-
(A) Angular momentum (B) Linear momentum
(C) Kinetic energy (D) Angular speed
3. A uniform rod AB of mass m and length l at rest on a smooth horizontal A
surface. An impulse P is applied to the end B. The time taken by the
rod to turn through a right angle is :-

2 m m
(A) (B)
P 3P
m  2 m P
(C) (D) B
12P 3P
4. An equilateral prism of mass m rests on a rough horizontal surface with coefficient of friction µ. A horizontal
force F is applied on the prism as shown in the figure. If the coefficient of friction is sufficiently high so
that the prism does not slide before toppling, then the minimum force required to topple the prism is-
F
mg mg
(A) (B)
3 4
a a
µ mg µ mg
(C) (D)
3 4 a
5. A uniform rod of mass M and length L lies radially on a disc rotating with angular speed  in a horizontal
plane about its axis. The rod does not slip on the disc and the centre of the rod is at a distance R from
the centre of the disc. Then the kinetic energy of the rod is- 

1  2 L2  L
2 1
(A) 2 m  R 


12  (B) m 2 R 2 R
 2

1
(C) m 2 L2 (D) None of these
24

6. A tube of length L is filled completely with an incompressible liquid of mass M and closed at both the ends.
The tube is then rotated in a horizontal plane about one of its ends with a uniform angular velocity  . The
force exerted by the liquid at the other end is :-
M 2 L M 2 L M 2 L2
(A) (B) M  2L (C) (D)
2 4 2

Page 65 of 99
Physics
7. Two point masses of 0.3 kg and 0.7 kg are fixed at the ends of a rod of length 1.4 m and of negligible
mass. The rod is set rotating about an axis perpendicular to its length with a uniform angular speed. The
point on the rod through which the axis should pass in order that the work required for rotation of the rod
is minimum, is located at a distance of :-
(A) 0.42 m from mass of 0.3 kg (B) 0.70 m from mass of 0.7 kg
(C) 0.98 m from mass of 0.3 kg (D) 0.98 m from mass of 0.7 kg
8. A sphere S rolls without slipping, moving with a constant speed on a plank P. The friction between the upper
surface of P and the sphere is sufficient to prevent slipping, while the lower surface of P is smooth and rests
on the ground. Initially, P is fixed to the ground by a pin T. If T is suddenly removed-
(A) S will begin to slip on P. 
S

(B) P will begin to move backwards. v = r


T r
P
(C) the speed of S will decrease and its angular velocity will increase.
(D) there will be no change in the motion of S and P will still be at rest.
9. A disc of mass M and radius R is rolling with angular speed  on a horizontalplane as shown. The mag-
nitude of angular momentum of the disc about the origin O is :–
y 
1
(A) MR 2  (B) MR  2
2
M
3 O x
(C) MR 2  (D) 2MR2 
2
Y

10. Two spheres each of mass M and radius R/2 are connected with a mass P
M M

2
Q
less rod of length 2R as shown in the-figure. What will be the moment

R/
of inertia of the system about an axis passing through the centre of one
of the spheres and perpendicular to the rod
Y’
21 2 5 5
(A) MR2 (B) M R 2 (C) M R 2 (D) MR2
5 5 2 21
11. A cord is wound over a cylinder of radius r and moment of inertia I. A mass m is attached to the free end
of the cord. The cylinder is free to rotate about its own horizontal axis. If mass m is released from rest, then
the velocity of the mass after it had fallen through a distance h will be-
1/2 1/2 1/2
 2mghr²   2mghr²   mghr² 
(A) (2gh)1/2 (B)   (C)  (D) 
I I  mr²  I  2mr² 

12. A solid sphere of radius R is placed on smooth horizontal surface. A horizontal force
‘F’ is applied at height ‘h’ from the lowest point. For the maximum acceleration F

of centre of mass, which is correct- R


h
(A) h = R
(B) h = 2R
Lowest Point
(C) h = 0
(D) No relation between h and R

Page 66 of 99
Physics
13. A solid sphere is placed on a horizontal plane. A horizontal impulse I is applied at a distance h above the central
line as shown in the figure. Soon after giving the impulse the sphere starts rolling.
The ratio h/R would be- I
h
1 2
(A) (B) C
R
2 5

1 1
(C) (D)
4 5
14. A ring of radius 3a is fixed rigidly on a table. A small ring whose mass is m
m
and radius a, rolls without slipping inside it as shown in the figure. The
A a
small ring is released from position A. When it reaches at the lowest point,
3a
the speed of the centre of the ring at that time would be-

(A) 2ga (B) 3ga (C) 6ga (D) 4ga


15. The moment of inertia of semicircular plate of radius R and mass M about axis AA' in its plane passing through
its centre is A
MR2 MR 2
(A) (B) cos 2  
2 4

MR 2 MR2 A'
(C) sin 2  (D)
4 4
16. The figure shows a uniform rod lying along the x-axis. The locus of all the
y
points lying on the xy-plane, about which the moment of inertia of the rod is
same as that about O is
(A) an ellipse (B) a circle x
(C) a parabola (D) a straight line o

17. A man can move on a horizontal plank supported symmetrically as shown. x=0

The variation of normal reaction on support A with distance x of the man A B

from the end of the plank is best represented by 1m 1m

N N N N

(A) (B) (C) (D)

x x x x
m
18. Find minimum height of obstacle so that the sphere can stay in equilibrium
R
R R
(A) (B) h
1  cos  1  sin 

(C) R 1  sin  (D) R 1  cos 

Page 67 of 99
Physics
19. A sphere is placed rotating with its centre initially at rest in a corner (A) (B)
as shown in figure (A) & (B). Coefficient of friction between all surfaces

1 fa
and the sphere is . Find the ratio of the frictional force f by ground
3 b

in situations (A) & (B)


(A) 1 (B) 9/10 (C) 10/9 (D) None
20. A hinged construction consists of three rhombus with the ratio of
sides 5:3:2. Vertex A3 moves in the horizontal direction at a velocity
v. Velocity of A2 is
v
A0 A1 A2 A3
(A) 2.5 v (B) 1.5 v
2
(C) v (D) 0.8 v
3
21. A disc of radius R is rolling purely on a flat horizontal surface, with a constant C
P
angular velocity. The angle between the velocity and acceleration vectors of
point P is
(A) Zero (B) 450
(C) 1350 (D) tan-1(1/2)
22. Portion AB of the wedge shown in figure is rough and BC is smooth. A
solid cylinder rolls without slipping from A to B. The ratio of translational A \
\ \\
\\\
\\\
kinetic energy to rotational kinetic energy, when the cylinder reaches point
\\
\\\ \ \
\\\
\\\

C is B

(A) 3/4 (B) 5 D C


AB=BC

(C) 7/5 (D) 8/3


23. A ring of mass m and radius R has three particles attached to the ring
m
as shown in the figure. The centre of the ring has a speed v0. The kinetic
2m m
energy of the system is : (slipping is absent)
(A) 6 mv02 (B) 12 mv02
(C) 4 mv02 (D) 8 mv02
24. A slender uniform rod of length  is balanced vertically at a point P on a horizontal surface having some friction.
If the top of the rod is displaced slightly to the right, the position of its centre of mass at the time when the
rod becomes horizontal
(A) lies at some point to the right of P (B) lies at some point to the left of P

(C) must be to the right of P (D) lies at P
2
25. A solid sphere with a velocity (of centre of mass) v and angular velocity  is gently placed on a rough horizontal
surface. The frictional force on the sphere
(A) must be forward (in direction of v) (B) must be backward (opposite to v)
(C) cannot be zero (D) none of the above

Page 68 of 99
Physics
0
26. A uniform circular disc placed on a rough horizontal surface has initially
v0
velocity v0 and an angular velocity  0 as shown in the figure. The disc comes

to rest after moving some distance in the direction of motion. Then v 0 is


r 0
(A) 1/2 (B) 1 (C) 3/2 (D) 2

SECTION - B MULTIPLE CORRECT QUESTIONS


1. ABCD is a square plate with centre O. The moments of inertia of the plate about the perpendicular axis
through O is I and about the axes 1, 2, 3 & 4 are I1, I2, I3 & I4 respectively. It follows that :

(A) I2 = I3 (B) I = I1 + I4 (C) I = I2 + I4 (D) I1 = I3

2. A rod of weight w is supported by two parallel knife edges A and B and is in equilibrium in a horizontal position.
The knives are at a distance d from each other. The centre of mass of the rod is at a distance x from A.
wx w (d  x )
(A) the normal reaction at A is (B) the normal reaction at A is
d d
wx w (d  x )
(C) the normal reaction at B is (D) the normal reaction at B is
d d
3. A block with a square base measuring axa and height h, is placed on an inclined plane. The coefficient of friction
is m. The angle of inclination ( θ ) of the plane is gradually increased. The block will
a a
(A) topple before sliding if   (B) topple before sliding if  
h h
a a
(C) slide before toppling if   (D) slide before toppling if  
h h
4. A body is in equilibrium under the influence of a number of forces. Each force has a different line of action. The
minimum number of forces required is
(A) 2, if their lines of action pass through the centre of mass of the body.
(B) 3, if their lines of action are not parallel.
(C) 3, if their lines of action are parallel.
(D) 4, if their lines of action are parallel and all the forces have the same magnitude.

Page 69 of 99
Physics
5. Four point masses are fastened to the corners of a frame of negligible
mass lying in the xy plane. Let  be the angular speed of rotation. Then
(A) rotational kinetic energy associated with a given angular speed
depends on the axis of rotation.
(B) rotational kinetic energy about y-axis is independent of m and its
value is Ma2  2.
(C) rotational kinetic energy about z-axis depends on m and its value is (Ma2 + mb2)  2.
(D) rotational kinetic energy about z-axis is independent of m and its value is Mb2  2.
6. A block of mass m moves on a horizontal rough surface with initial velocity v. The height of the centre of mass
of the block is h from the surface. Consider a point A on the surface.
(A) angular momentum about A is mvh initially
(B) the velocity of the block decreases at time passes.
(C) torque of the forces acting on block is zero about A
(D) angular mometum is not conserved about A.
7. A ring rolls without slipping on the ground. Its centre C moves with a constant speed u. P is any point on the
ring. The speed of P with respect to the ground is n.
( A < n < 2u
) 0

(B) n = u, if CP is horizontal
(C) n = u, if CP makes an angle of 30° with the horizontal and P is below the horizontal level of C.
(D) n = 2u , if CP is horizontal
8. A yo-yo is resting on a rough horizontal table. Forces F1, F2 and F3 are applied separately as shown. The
correct statement is
(A) when F3 is applied the centre of mass will move to the right.
(B) when F2 is applied the centre of mass will move to the left.
(C) when F1 is applied the centre of mass will move to the right.
(D) when F2 is applied the centre of mass will move to the right.
9. A disc of circumference s is at rest at a point A on a horizontal surface when a constant horizontal force begins
to act on its centre. Between A and B there is sufficient friction to prevent slipping, and the surface is smooth to
the right of B. AB = s. The disc moves from A to B in time T. To the right of B,

(A) the angular acceleration of the disc will disappear, linear acceleration will remain unchanged
(B) linear acceleration of the disc will increase
(C) the disc will make one rotation in time T/2
(D) the disc will cover a distance greater than s in further time T.

Page 70 of 99
Physics
10. A plank with a uniform sphere placed on it, rests on a smooth horizontal plane.
Plank is pulled to right by a constant force F. If the sphere does not slip over
the plank.
(A) acceleration of centre of sphere is less than that of the plank
(B) acceleration of centre of sphere is greater than the plank because friction acts rightward on the sphere
(C) acceleration of the centre of sphere may be towards left.
(D) acceleration of the centre of sphere relative to plank may be greater than that of the plank relative to floor.
11. A hollow sphere of radius R and mass m is fully filled with water of mass m. It is rolled down a horizontal plane
such that its centre of mass moves with a velocity v. If it purely rolls
5
(A) Kinetic energy of the sphere is mv2
6
4
(B) Kinetic energy of the sphere is mv2
5
8
(C) Angular momentum of the sphere about a fixed point on ground is mvR
3
14
(D) Angular momentum of the sphere about a fixed point on ground is mvR
5
12. In the figure shown, the plank is being pulled to the right with a constant speed v. If the cylinder does not slip
then:
(A) the speed of the centre of mass of the cylinder is 2v.
(B) the speed of the centre of mass of the cylinder is zero.
(C) the angular velocity of the cylinder is v/R.
(D) the angular velocity of the cylinder is zero.
13. A uniform disc is rolling on a horizontal surface. At a certain instant B is the
point of contact and A is at height 2R from ground, where R is radius of disc.
(A) The magnitude of the angular momentum of the disc about B is thrice that about A.
(B) The angular momentum of the disc about A is anticlockwise.
(C) The angular momentum of the disc about B is clockwise
(D) The angular momentum of the disc about A is equal to that about B.
14. If a cylinder is rolling down the incline with sliding.
(A) after some time it may start pure rolling
(B) after sometime it will start pure rolling
(C) it may be possible that it will never start pure rolling
(D) none of these
15. Which of the following statements are correct.
(A) friction acting on a cylinder without sliding on an inclined surface is always upward along the incline
irrespective of any external force acting on it.
(B) friction acting on a cylinder without sliding on an inclined surface is may be upward may be downwards
depending on the external force acting on it.
(C) friction acting on a cylinder rolling without sliding may be zero depending on the external force acting on it.
(D) nothing can be said exactly about it as it depends on the friction coefficient on inclined plane.

Page 71 of 99
Physics
16. Choose the correct alternative(s)
3v 02
(A) The linear distance moved by the centre of mass before the ring starts rolling is
8g
3 2
(B) The net work done by friction force is – mv 0
8
mv02
(C) The loss is kinetic energy of the ring is
4
mv 02
(D) The gain in rotational kinetic energy is +
8

17. Consider a sphere of mass ‘m’ radius ‘R’ doing pure rolling motion on a rough surface having velocity v 0 as
shown in the Fig. It makes an elastic impact with the smooth wall and moves back and starts pure rolling after
some time again.
(A) Change in angular momentum about ‘O’ in the entire motion equals 2mv0R in magnitude.
(B) Moment of impulse provided by the wall during impact about O equals 2mv0R in magnitude.
3
(C) Final velocity of ball will be v0
7
3
(D) Final velocity of ball will be – v0
7
SECTION - C COMPREHENSION TYPE
Comprehension #1
h
A solid sphere is kept over a smooth surface as shown is figure. It is hit by a cue C

at height h above the centre C.

R R
1. In case 1, h = and in case 2, h = . Suppose in case 1 the sphere acquires a total kinetic energy K1
4 2
and in case 2 total kinetic energy is K2. Then :-
(Note: That in both the cases, sphere is hit by the same impulse)
(A) K1 = K2 (B) K1 > K2 (C) K1 < K2 (D) data is insufficient
2. If the surface is rough, then after hitting the sphere, in which case the force of friction is in forward direction:-
(A) in case 1 (B) in case 2 (C) in both the cases (D) in none of the case
Comprehension #2
In rotational motion if angular acceleration (or retardation) is constant we can apply equations of motion

ω = ω 0 ± at etc. Here a = I .
1. A solid sphere of mass 5 kg and radius 1 m after rotating with angular speed
ω 0 = 40 rad/s is placed between two smooth walls on a rough ground. Distance
between the walls is slightly greater than the diameter of the sphere. Coefficient
of friction between the sphere and the ground is µ = 0.1. Sphere will stop
rotating after time t = ............. s :-
(A) 8 (B) 12 (C) 20 (D) 16

Page 72 of 99
Physics
Comprehension #3
A solid sphere is rolling without slipping on rough ground as shown in figure. v

It collides elastically with an identical another sphere at rest. There is no friction
between the two spheres. Radius of each sphere is R and mass is m.
1. Linear velocity of first sphere after it again starts rolling without slipping is :-
2 2 7 7
(A) wR (B) wR (C) wR (D) wR
5 7 10 5

2. What is the net angular impulse imparted to second sphere by the external forces ?
2 5 2 7
(A) mRv (B) mRv (C) mRv (D) mRv
7 7 5 10

Comprehension #4
C
A small sphere of mass 1 kg is rolling without slipping on a stationary base with
1m
200 v
linear speed v = m/s. It leaves the inclined plane at point C. 30°
7 A B
1. Find its linear speed at point C :-
100 50 100 200
(A) m/s (B) m/s (C) m/s (D) m/s
7 7 35 35
2. Find ratio of rotational and translational kinetic energy of the sphere when it strikes the ground after leaving
from point C :-
2 2 1 1
(A) (B) (C) (D)
5 3 6 2
Comprehension #5
A rod AB of length 2 m and mass 2 kg is lying on smooth horizontal x- y plane with its centre at origin O as shown
figure. An impulse J of magnitude 10 N-s is applied perpendicular to AB at A.
y
A
J

O x

1. The distance of point P from centre of the rod which is at rest just after the impact is :-
2 1 1 1
(A) m (B) m (C) m (D) m
3 3 2 4

2. Co-ordinates of point A of the rod after time t = s will be :-
45
  3 1   3
 3     1 1  1 1 
(A)  9  2  m, 2 m  (B)   4 m , 2 m   (C)  6  2  m, 2 m  (D)  2 m, 2 m 
        

Page 73 of 99
Physics
Comprehension #6
When a force F is applied on a block of mass m resting on a horizontal surface then there are two possibilities,
either block moves by translation or it moves by toppling. If the surface is smooth then the block always
translates but on a rough surface it topples only when the torque of the applied force F is greater than the
torque of mg about a point in contact with the ground.
F
m h

A
a
When the force F is applied the body may topple about A or it may translate.
1. When the block topples about A, the normal force :-
(A) passes through centre of mass
(B) is zero
(C) shifts to the right and passes through rightmost edge containing A
(D) is zero if the surface is smooth
2. If the block be a cube of edge a and µ = 0.2 then :-
(A) the body will translate (B) the body will topple
(C) the body may translate or topple (D) none of the above
3. If the block is a cube of edge a and µ = 0.6 then :-
(A) the body will translate (B) the body will topple
(C) the body first translates and then topples (D) none of the above
Comprehension #7
In figure, the winch is mounted on an axle, and the 6-sided nut is welded to the winch. By turning the nut
with a wrench, a person can rotate the winch. For instance, turning the nut clockwise lifts the block off the
ground, because more and more rope gets wrapped around the winch.

nut

winch
person grips
wrench here

Block
Wrench turns winch clockwise

Three students agree that using a longer wrench makes it easier to turn the winch. But they disagree about
why. All three students are talking about the case where the winch is used, over a 10 s time interval, to
lift the block one metre off the ground.
Student 1 By using a longer wrench, the person decreases the average force he must exert on the wrench,
in order to lift the block one metre in 10 s.
Student 2 : Using a longer wrench reduces the work done by the person as he uses the winch to lift the
block 1m in 10s.
Student 3 : Using a longer wrench reduces the power that the person must exert to lift the block 1m in
10s.

Page 74 of 99
Physics
1. Student 1 is :-
(A) correct, because the torque that the wrench must exert to lift the block doesn't depend on the wrench's
length
(B) correct, because using a longer wrench decreases the torque it must exert on the winch
(C) incorrect, because the torque that the wrench must exert to lift the block doesn't depend on the
wrench's length
(D) Incorrect, because using a longer wrench decreases the torque it must exert on the winch.
2. Which of the following is true about student 2 and 3 :-
(A) Student 2 and 3 are both correct
(B) Student 2 is correct, but student 3 is incorrect
(C) Student 3 is correct, but student 2 is incorrect
(D) Student 2 and 3 are both incorrect
3. If several wrenches all apply the same torque to a nut, which graph best expresses the relationship between
the force the person must apply to the wrench, and the length of the wrench :-
force

force
force
force

(1) (2) (3) (4)

length length length length

(A) 1 (B) 2 (C) 3 (D) 4


Comprehension #8
In the figure shown a plank of mass m is lying at rest on a smooth horizontal surface. A disc of same mass m
and radius r is rotated to an angular speed w0 and then gently placed on the plank. If we consider the plank and
the disc as a system then frictional force between them is an internal force. Momentum of the system changes
due to external force only. It is found that finally slipping cease, and 50% of total kinetic energy of the system is
lost. Assume that plank is long enough. m is coefficient of friction between disc and plank.
m, r

0

1. Final velocity of the plank is

r0 r0 r0 r0


(A) (B) (C) (D)
4 10 2 2 10

2. Time when slipping ceases

r0 r0 r0 r0


(A) 2 g (B) (C) 4 g (D)
10 g 2 10 g

3. Magnitude of the change in angular momentum of disc about centre of mass of disc
3 1 1
(A) mr 2  0 (B) mr 2  0 (C) zero (D) mr 2  0
4 4 2

Page 75 of 99
Physics
4. Distance moved by the plank from the placing of disc on the plank till the slipping ceases between disc and
plank
r 2  20 r 2 20 r 2 20
(A) r 0 (B) 8 g (C) 32 g (D) 200 g
2 2

16 g

Comprehension #9
A ring of mass M and radius R sliding with a velocity v0 suddenly enters into rough surface where the
coefficient of friction is  , as shown in figure.

v0

Rough()

1. Choose the correct statement(s)


(A) As the ring enters on the rough surface, the limiting friction force acts on it
(B) The direction of friction is opposite to the direction of motion
(C) The friction force accelerates the ring in the clockwise sense about its centre of mass
(D) As the ring enters on the rough surface it starts rolling
2. Choose the correct statement(s)
(A) The momentum of the ring is conserved
(B) The angular momentum of the ring is conserved about its centre of mass
(C) The angular momentum of the ring conserved about any point on the horizontal surface
(D) The mechanical energy of the ring is conserved
3. Choose the correct statement(s) :–
(A) The ring starts its rolling motion when the centre of mass stationary
(B) The ring starts rolling motion when the point of contact becomes stationary
v0
(C) The time after which the ring starts rolling is 2 g

v0
(D) The rolling velocity is
2
4. Choose the correct alternative(s)
3v 2
(A) The linear distance moved by the centre of mass before the ring starts rolling is 8 g0

3
(B) The net work done by friction force is – mv02
8

m v 20
(C) The loss in kinetic energy of the ring is
4

m v 20
(D) The gain in rotational kinetic energy is +
8

Page 76 of 99
Physics
SECTION - D ASSERTION - REASON TYPE
In each of the following questions, a Statement of Assertion (A) is given followed by a corresponding Statement
of Reason (R) just below it . Of the Statements mark the correct answer as
(A) Statement–1 is True, Statement–2 is True ; Statement–2 is a correct explanation for Statement–1
(B) Statement–1 is True, Statement–2 is True ; Statement–2 is not a correct explanation for Statement–1
(C) Statement–1 is True, Statement–2 is False.
(D) Statement–1 is False, Statement–2 is True.
1. Statement –1 : A rigid disc rolls without slipping on a fixed rough horizontal surface with uniform angular
velocity. Then the acceleration of lowest point on the disc is zero.
Statement –2 : For a rigid disc rolling without slipping on a fixed rough horizontal surface, the velocity
of the lowest point on the disc is always zero.
2. Statement –1 : The torque can be applied only about two points.
(i) centre of mass and (ii) point about which the body is rolling.
Statement –2 : The equation a = r  can always be applied in case of rolling.
3. Statement –1 : In case of rolling friction force can in forward and backward direction both.
Statement –2 : The angular momentum of a system will be conserved only about that point about which
external angular impulse is zero.
4. Statement –1 : For the purpose of calculation of moment of inertia, a body’s mass can be thought to be
concentrated at its centre of mass.
Statement –2 : Moment of inertia is a measure of how the mass is distributed about a certain axis.
5. Statement –1 : If a body (ball) is rolling on a surface without slipping, no frictional force acts on it.
Statement –2 : In the case of rolling without slipping point of contacts are relatively at rest.
    
6. Statement –1 : Torque   acting on a rigid body is defined as   A  L, A is a constant vector and

L is the angular momentum of the body. The magnitude of the angular momentum of
the body remains same.

Statement –2 :  is perpendicular to L and also perpendicular to  , hence torque does not deliver any
power to the body.
7. Statement –1 : The moment of inertia of a rigid body is not unique, about a given axis.
Statement –2 : The moment of inertia of a rigid body depends on axis about which it has to be calculated.
8. Statement –1 : A sphere rolling on a rough horizontal surface with constant velocity then it start going
up on a smooth inclined plane. Rotational KE of sphere decreases continuously on
horizontal and inclined surface.
Statement –2 : Rotational KE decreases if torque due to friction opposes angular velocity of sphere.
9. Statement –1 : A disc is rolling on an inclined plane without slipping. The velocity of centre of mass
is v. These others points on the disc lies on a circular arc having same speed as centre
of mass.
Statement –2 : When a disc is rolling on an inclined plane. The magnitude of velocities of all the point from
the contact point is same, having distance equal to radius r.

Page 77 of 99
Physics
10. Statement –1 : A non-uniform sphere is placed such that its centre is origin of coordinate system. If
Ix and Iy be moment of inertia about x axis and y axis respectively then moment of inertia
about z axis is Ix + Iy.
Statement –2 : According to perpendicular axis theory Iz = Ix + Iy when object is lying in x-y plane.
11. Statement –1 : A sphere is performing pure rolling on a rough horizontal surface with constant angular
velocity. Frictional force acting on the sphere is zero.
Statement –2 : Velocity of contact point is zero.
SECTION - E MATRIX MATCH TYPE
1. In each situation of column-I, a uniform disc of mass m and radius R rolls on a rough fixed horizontal surface
a s s h o w n . A t t = 0 ( i
 0 and velocity of centre of mass of disc is
n i t i a l l y ) t h e a n g u l a r v e l o c i t y o f d i s c i s

v0 (in horizontal direction). The relation between v0 and  0 for each situation and also initial sense of rotation
is given for each situation in column-I. Then match the column the Statement in column-I with the corresponding
results in column-II.
Column I Column II
0
v0
(A) (v0 > R 0) (p) The angular momentum of disc about point A (as shown in figure) remains
A
conserved

0
v0
(B) (v0 > R 0) (q) The kinetic energy of disc after it starts rolling without
A
slipping is less than its initial kinetic energy

0
v0
(C) (v0 < R 0) (r) In the duration disc rolls with slipping, the friction
A
acts on disc towards left.
0
(D) v0
(v0 < R 0)
(s) In the duration disc rolls with slipping, the friction
A acts on disc for some time to right and for some
time to left.
2. Four rods of equal length l and mass m each form a square as shown in figure. Moment of inertia about four
axes 1, 2,3 and 4 are say I1, I2, I3 and I4.
Column I Column II
4
(A) I1 (p) 4 ml 2 3
3

2 2 1
(B) I2 (q) ml
3

1 2 2
(C) I3 (r) ml
2
(D) I4 (s) None

Page 78 of 99
Physics
3. Column I Column II
(A) In pure rolling work done (p) is always zero
by friction (q) may be zero
(B) In forward slipping work done (r) is negative
by friction (s) is positive
(C) In backward slipping work (t) may be negative
done by friction (u) may be positive
4. A disc with linear velocity v and angular velocity w is placed on rough ground. Suppose a and a be the magnitudes
of linear and angular acceleration due to friction. Then :-
Column I Column II
(A) When v = Rw (p) a = Ra (a ¹ 0)
v

R
(B) When v = (q) a > Ra
2
(C) When v = 2Rw (r) a < Ra
(s) None
5. A solid sphere is rotating about an axis as shown in figure. An insect follows the dotted path on the circumference
of sphere as shown.
Column I Column II
(A) Moment of inertia (p) will remain constant
Insect
(B) Angular velocity (q) will first increase
(C) Angular momentum then decrease
(D) Rotational kinetic energy (r) will first decrease
then increase
(s) will continuously decrease
(t) will continuously increase
(u) data is insufficient
6. In the adjacent figure a uniform rigid body of mass m and radius R is kept at F
rest on a rough horizontal surface. A constant horizontal force F is applied at the
top most point of the body. The body starts rolling without slipping. Different acm
shapes of bodies are given in the column I and based on this problem some R
physical quantities related to them are given in column II.
Rough
Column I Column II
(A) Solid sphere (p) Friction force is zero
(B) Ring (q) Magnitude offriction force is
(C) Hollow sphere maximum
(D) Disc (r) Acceleration of C. O. M.
is 4F/3 m
(s) Magnitude of friction force is F/5

Page 79 of 99
Physics
SECTION - F SUBJECTIVE QUESTIONS
1. A rectangular rigid fixed block has a long horizontal edge. A solid homogeneous
cylinder of radius R is placed horizontally at rest with its length parallel to
the edge such that the axis of the cylinder and the edge of the block are R

in the same vertical plane as shown in figure. There is sufficient friction present
at the edge, so that a very small displacement causes the cylinder to roll of
the edge without slipping. Determine :

(i) The angle θ C through which the cylinder rotates before it leaves contact with the edge.

(ii) The speed of the centre of mass of the cylinder before leaving contact with the edge and
(iii) The ratio of the translational to rotational kinetic energies of the cylinder when its centre of mass is in
horizontal line with the edge.
2. A uniform rod of length 4l and mass m is free to rotate about a horizontal

axis passing through a point distance l from its one end. When the

rod is horizontal, its angular velocity is  as shown in figure. Calculate
(i) reaction of axis at this instant,
(ii) acceleration of centre of mass of the rod at this instant,
(iii) reaction of axis and acceleration of centre of mass of the rod when rod becomes vertical for the first time.
(iv) minimum value of  so that centre of rod can complete circular motion.
3. A semi circular track of radius R = 62.5 cm is cut in a block. Mass of block, having
m
track,is M = 1 kg and rests over a smooth horizontal floor. A cylinder of radius r = 10
R
cm and mass m = 0.5 kg is hanging by thread such that axis of cylinder and track are
in same level and surface of cylinder is in contact with the track as shown in figure. M
When the thread is burnt, cylinder starts to move down the track. Sufficient friction
exists between surface of cylinder and track, so that cylinder does not slip. Calculate
velocity of axis of cylinder and velocity of the block when it reaches bottom of the
t r a c k . A l s o f i n d f o r c e a p
2
)p l i e d b y b l o c k o n t h e f l o o r a t t h e m o m e n t . ( g = 1 0 m / s

4. A thin rod is passing through the centre of a sphere. The rod is fixed to a vertical 

axis and the sphere is made to roll on a surface with friction. The radius of the 
sphere is r, the mass is m and the length of the rod is l. The rod is rotating with
an angular velocity 0 . Find the energy of the sphere in terms of 0 , m, l and
r. Assume the rod to be of negligible mass.

Page 80 of 99
Physics
5. A man and a woman skate towards each other on smooth ice, but in
M
parallel lines. The distance between the lines is l. The mass of the man is V
M and that of the woman is m. The velocity of the man is given by V and
that of the woman by v. The woman holds a stick of length l and negli- 
gible mass. The stick is directed normal to the direction of motion as
shown in the figure. When the couple passes each other, the man grasps v
the stick and the couple move together, each of them holding different W
ends of the stick.
(i) What is the angular velocity of the rod after the couple begin moving together ?
(ii) The couple start moving towards each other by pulling the stick until the distance between them is l0(l0 < l).
What is the velocity of the centre of mass now ?
(iii) What is the angular velocity of the couple now ?
(iv) What is the work done by the couple as they move from l to l0 ?
6. A square frame is formed by four rods, each of length l = 60 cm. Mass of two
rods AB and BC is m = 25/18 kg each while that of rods AD and CD
is 2kg each. The frame is free to rotate about a fixed horizontal axis A
a
B

passing through its geometric centre O shown in figure. A spring is placed


on the rod AB at a distance a = 15 cm from B. The spring is held vertical O

and a block is placed on upper end of the spring so that rod AB is horizontal. D C
60cm

(i) Calculate mass M of the block,


(ii) If the spring is initially compressed by connecting a thread between its ends and energy stored in it is 76.5
joule, calculate velocity with which block bounces up when the thread is burnt.
7. A small ring of mass m is threaded on a horizontal smooth rod which is rotating about its end with constant
angular velocity w. The ring is initially located at the axis of rotation. When the distance of the ring from the axis
becomes r, then find the power required to rotate the system with same angular velocity.

Page 81 of 99
Physics
EXERCISE - 5 PREVIOUS YEAR QUESTIONS
SECTION - A MAINS LEVEL SINGLE CORRECT TYPE
1. Initial angular velocity of a circular disc of mass M is  1. Then two small spheres of mass m are attached gently
to two diametrically opposite points on the edge of the disc. What is the final angular velocity of the disc ?

 M m   M m   M   M 
(A)   1 (B)   1 (C)   1 (D)   1
 M   m   M  4m   M  2m 

2. Moment of inertia of a circular wire of mass M and radius R about its diameter is-
(A) MR2/2 (B) MR2 (C) 2MR2 (D) MR2/4
3. A particle of mass m moves along line PC with velocity v as shown.What is the angular momentum of the
particle about O ?
C

P r

(A) mvL (B) mvl (C) mvr (D) zero


4. A circular disc X of radius R is made from an iron plate of thickness t and another disc Y of radius 4R is made
from an iron plate of thickness t/4. Then the relation between the moment of inertia IX and IY is-
(A) IY = 32 IX (B) IY = 16IX (C) IY = IX (D) IY = 64 IX
5. A particle performing uniform circular motion has angular momentum L. If its angular frequency is doubled and
its kinetic energy halved, then the new angular momentum is-
L L
(A) (B) 2L (C) 4L (D)
4 2

6. Let F be the force acting on particle having position vector  
r and  be the torque of this force about the
origin. Then-
 
(A)   
r .  = 0 and F .  ¹ 0 (B)   
r .  ¹ 0 and F .  = 0

 
(C)   
r .  ¹ 0 and F .  ¹ 0 (D)   
r .  = 0 and F .  = 0

7. Which of the following statements is false for a particle moving in a circle with a constant angular
speed ?
(A) The velocity vector is tangent to the circle
(B) The acceleration vector is tangent to the circle
(C) the acceleration vector points to the centre of the circle
(D)The velocity and acceleration vectors are perpendicular to each other

Page 82 of 99
Physics
8. A solid sphere is rotating in free space. If the radius of the sphere is increased keeping mass same which one of
the following will not be affected ?
(A) moment of inertia (B) Angular momentum (C) Angular velocity (D) Rotational kinetic anergy
9. One solid sphere A and another hollow sphere B are of same mass and same outer radii. Their moment of
inertia about their diameters are respectively IA and IB such that-

IA d
(A) IA = IB (B) IA > IB (C) IA < IB (D) = A
IB dB

where dA and dB are their densities.


10. An annular ring with inner and outer radii R1 and R2 is rolling without slipping with a uniform angular speed. The

ratio of the forces experienced by the two particles situated on the inner and outer parts of the ring F1 is-
F2
2
R2 R  R1
(A) (B)  1  (C) 1 (D) R
R1  R2  2

11. The moment of inertia of uniform semicircular disc of mass M and radius r about a line perpendicular to the
plane of the disc through the centre is-

1 2 1
(A) Mr2 (B) Mr2 (C) Mr2 (D) Mr2
4 5 2

12. A spherical ball of mass 20 kg is stationary at the top of a hill of height 100 m. It rolls down a smooth surface
to the ground, then climbs up another hill of height 30 m and finally rolls down to a horizontal base at a height
of 20 m above the ground. The velocity attained by the ball is-

5
(A) 40 m/s (B) 20 m/s (C) 10 m/s (D) 10 30 m/s
7

13. A coin is placed on a horizontal platform which undergoes vertical simple harmonic motion of angular frequency
w. The amplitude of oscillation is gradually increased. The coin will leave contact with the platform for the first
time :
(A) at the mean position of the platform (B) for an amplitude of g/w2
(C) for an amplitude of g2/w2 (D) at the highest position of the platform

14. Four point masses, each of value m, are placed at the corners of a square ABCD of side l. The moment of inertia
of this system about an axis passing through A and parallel to BD is

(A) 2ml2 (B) 3 ml2 (C) 3ml2 (D) ml2

Page 83 of 99
Physics
15. A force of –F k̂ acts on O, the origin of the co-ordinate system. The torque about the point
(1, –1) is :
z
(A) F ˆi  ˆj 

(B) – F ˆi  ˆj 
O y
(C) F ˆi  ˆj 

(D) – F ˆi – ˆj  x

16. A thin circular ring of mass m and radius R is rotating about its axis with a constant angular velocity  . Two
objects each of mass M are attached gently to the opposite ends of a diameter of the ring. The ring now
rotates with an angular velocity  ' = :

(m  2M ) (m  2M ) m m
(A) (B) (C) (D)
m (m  2M ) (m  M ) (m  2M )

17. For the given uniform square lamina ABCD, whose centre is O :

F
D • C
•O

A • B
E

(A) 2 IAC = IEF (B) IAD = 3IEF (C) IAC = IEF (D) IAC = 2 IEF
18. A round uniform body of radius R, mass M and moment of inertia I, rolls down (without slipping) an inclined
plane making an angle q with the horizontal. Then its acceleration is

g sin  g sin  g sin  g sin 


(A) 2 (B) 2
(C) 2 (D)
1  I / MR 1  MR / I 1  I / MR 1  MR 2 / I

19. Angular momentum of the particle rotating with a central force is constant due to-
(A) constant force (B) constant linear momentum
(C) zero torque (D) constant torque
20. Consider a uniform square plat of side ‘a’ and mass ‘m’ the moment of inertia of this plate about an axis
perpendicular to its plane and passing through one of its corners is

5 1 7 2
(A) ma2 (B) ma2 (C) ma2 (D) ma2
6 12 12 3

21. A thin uniform rod of length l and mass m is swinging freely about a horizontal axis passing through its end. Its
maximum angular speed is  . Its centre of mass rises to a maximum height of:

1 l 2 2 1 l 2 2 1 l 2 2 1 l
(A) 2 g (B) 6 g (C) 3 g (D) 6 g

Page 84 of 99
Physics
22. A small particle of mass m is projected at an angle  with the x-axis with an initial velocity v0 in the x-y plane
v 0 sin 
as shown in the figure. At a time t < g
, the angular momentum of the particle is:

1 1
(A) mg v0 t2 cos  i (B) – mg v0 t2 cos  j (C) mg v0 t cos  k (D) – mg v0 t2 cos  k
2 2

Where i , j and k are unit vectors along x, y and z-axis respectively..

23. A pulley of radius 2 m is rotated about its axis by a force F = (20t – 5t2) newton (where t is measured in seconds)
applied tangentially. If the moment of inertia of the pulley about its axis of rotation is 10 kg m2, the number
of rotations made by the pulley before its direction of motion it reversed, is :-

(A) more than 6 but less than 9 (B) more than 9

(C) less than 3 (D) more than 3 but less than 6


24. A thin horizontal circular disc is rotating about a vertical axis passing through its centre. An insect is at rest at
a point near the rim of the disc. The insect now moves along a diameter of the disc to reach its other end.
Euring the fjourney of the insect, then angular speed of the disc :-
(A) continuously increases (B) first increases and then decreases
(C) remains unchanged (D) continuously decreses
25. A particle of mass 'm' is projected with a velocity v making an angle of 30° with the horizontal. The
magnitude of angular momentum of the projectile about the point of projection when the particle is at its
maximum height 'h' is :-

3 mv 2 mv 3 3 mv 3
(A) 2 g
(B) zero (C) 2g
(D) 16 g

Page 85 of 99
Physics
SECTION - B ADVANCED LEVEL
SINGLE CORRECT QUESTIONS
1. Let I be the moment of inertia of a uniform square plate about an axis AB that C'
A'
D

passes through its centre and is parallel to two of its sides. CD is a line in the 
A B
plane of the plate that passes through the centre of the plate and makes an angle θ with
AB. The moment of inertia of the plate about the axis CD is then equal to:-
C D'
B'


(A) I (B) I sin2 θ (C) I cos2 θ (D) I cos2  2 
 

2. A cubical block of side 'a' moving with velocity v on a horizontal smooth plane as shown. It hits a ridge
at point O. The angular speed of the block after it hits O is :-

3v 3v
(A) (B) a
4a 2a
M v
3 O
(C) (D) zero
2a

3. A smooth sphere A is moving on a frictionless horizontal plane with angular velocity  and centre of mass
velocity v. It collides elastically and head on with an identical sphere B at rest. Neglect friction everywhere.
After the collision their angular speed are  A and  B respectively. Then :-
(A)  A <  B (B)  A =  B (C)  A =  (D)  B = 
4. A disc of mass M and radius R is rolling with angular speed  on a horizontal plane as shown. The magnitude
of angular momentum of the disc about the origin O is :-
y
1 
(A)  2  MR  (B) MR2 
2

  
M
3
(C)  2  MR  (D) 2MR2 
2
O x
 

5. An equilateral triangle ABC formed from a uniform wire has two small
identical beads initially located at A. The triangle is set rotating about A
the vertical axis AO. Then the beads are released from rest simultaneously g
and allowed to slide down, one along AB and other along AC as shown.
Neglecting frictional effects, the quantities that are conserved as beads
slides down are
(A) angular velocity and total energy (kinetic and potential) B C
O
(B) total angular momentum and total energy
(C) angular velocity and moment of inertia about the axis of rotation
(D) total angular momentum and moment of inertia about the axis of rotation

Page 86 of 99
Physics
6. A cubical block of side L rests on a rough horizontal surface with
F
coefficient of friction µ. A horizontal force F is applied on the block as
shown. If the coefficient of friction is sufficiently high, so that the block does L
not slide before toppling, the minimum force required to topple the block
is :-

mg mg
(A) infinitesimal (B) (C) (D) mg (1 – µ)
4 2

7. A thin wire of length L and uniform linear mass density r is bent into a circular loop with centre at O as
shown. The moment of inertia of the loop about the axis XX' is :-
X X'
90°
L 3
L 3
(A) (B)
8 2 16 2 O

5 L3 3 L3
(C) (D)
16 2 8 2

8. One quarter section is cut from a uniform circular disc of radius R. This section has
a mass M. It is made to rotate about a line perpendicular to its plane and passing
through the centre of the original disc. Its moment of inertia about the axis of rotation
is :-

1 1
(A) MR 2 (B) MR 2
2 4

1
(C) MR 2 (D) 2 MR 2
8

9. A cylinder rolls up an inclined plane and reaches some height and then rolls down (without slipping throughout
these motions). The directions of the frictional force acting on the cylinder are :-
(A) up the incline while ascending and down the incline while descending
(B) up the incline while ascending as well as descending
(C) down the incline while ascending and up the incline while descending
(D) down the incline while ascending as well as descending.
10. A circular platform is free to rotate in a horizontal plane about a vertical axis passing through its centre. A
tortoise is sitting at the edge of the platform. Now the platform is given an angular velocity  0. When the
tortoise move along a chord of the platform with a constant velocity (with respect to the platform). The angular
velocity of the platform  (t) will vary with time t as :-

(t) (t) (t) (t)


0 0 0 0
(A) (B) (C) (D)

t t t t

Page 87 of 99
Physics
11. Consider a body, shown in figure, consisting of two identical balls, each of mass M connected by a light
rigid rod. If an impulse J = Mv is imparted to the body at one of its end, what would be its angular velocity

L
M M

J=Mv

v 2v v v
(A) (B) (C) (D)
L L 3L 4L

12. A particle undergoes uniform circular motion. About which point on the plane of the circle, will the angular
momentum of the particle remain conserved ?
(A) Centre of circle (B) On the circumference of the circle
(C) Inside the circle (D) Outside the circle
13. A disc is rolling (with slipping) on a horizontal surface. C is its centre and Q and P are two points equidistant
from C. Let vP, vQ and vC be the magnitude of velocities of points P, Q and C respectively, then :-

C
P

1
(A) vQ > vC > vP (B) vQ < vC < vP (C) vQ = vP, vC = v (D) vQ < vC > vP
2 P

14. A child is standing with folded hands at the centre of a platform rotating about its central axis. The kinetic
energy of the system is K and moment of inertia is I. The child now stretches his arms so that the moment
of inertia of the system doubles. The kinetic energy of the system now is :-

K K
(A) 2K (B) (C) (D) 4K
2 4

15. A particle moves in a circular path with decreasing speed. Choose the correct Statement :
(A) Angular momentum remains constant

(B) Acceleration ( a ) is towards the centre
(C) Particle moves in a spiral path with decreasing radius
(D) The direction of angular momentum remains constant

Page 88 of 99
Physics
R
R
16. From a circular disc of radius R and mass 9M, a small disc of radius is 3
3 2R
3
O
removed from the disc. The moment of inertia of the remaining disc about R

an axis perpendicular to the plane of the disc and passing through O is :-

40 37
(A) 4MR2 (B) MR2 (C) 10MR2 (D) MR2
9 9

17. A solid sphere of radius R has moment of inertia I about its geometrical axis. If
it is melted into a disc of radius r and thickness t. If it's moment of inertia about the r
tangential axis (which is perpendicular to plane of the disc), is also equal to I, then
the value of r is equal to :-

2 2 3 3
(A) R (B) R (C) R (D) R
15 5 15 15

18. A small object of uniform density rolls up a curved surface with an initial velocity v. It reaches up to a maximum
3v 2
height of 4g with respect to the initial position. The object is :-

(A) ring (B) solid sphere (C) hollow sphere (D) disc

19. A block of base 10 cm × 10 cm and height 15 cm is kept on an inclined plane. The coefficient of friction
between them is 3 . The inclination θ of this inclined plane from the horizontal plane is gradually increased
from 0°. Then :-
(A) at θ =30°, the block will start sliding down the plane
(B) the block will remain at rest on the plane up to certain θ and then it will topple
(C) at θ =60°, the block will start sliding down the plane and continue to do so at higher angles
(D) at θ =60°, the block will start sliding down the plane and on further increasing θ , it will topple at certain θ
20. If the resultant of the external forces acting on a system of particles is zero, then from an inertial frame, one can
surely say that
(A) linear momentum of the system does not change in time
(B) kinetic energy of the system does not change in time
(C) angular momentum of the system does not change in time
(D) potential energy of the system does not change in time

Page 89 of 99
Physics
MULTIPLE CORRECT QUESTIONS
  
1. The torque  on a body about a given point is found to be equal to A × L , where A is a constant vector

and L is the angular momentum of the body about that point. From this it follows that :-

dL 
(A) is perpendicular to L at all instants of time
dt
 
(B) the component of L in the direction of A does not change with time

(C) the magnitude of L does not change with time

(D) L does not change with time
2. A solid sphere is in pure rolling motion on an inclined surface having
inclination θ :-
(A) frictional force acting on sphere is ƒ = µ mgcos θ
(B) ƒ is disspative force 
(C) friction will increase its angular velocity and decreases its linear velocity
(D) if θ decrease, friction will decrease
3. A ball moves over a fixed track as shown in the figure. From A to B the ball rolls without slipping. If surface
BC is frictionless and KA, KB and KC are kinetic energy of the ball at A, B and C respectively, then:-
A C

hA hC
B

(A) hA > hC; KB > KC (B) hA > hC; KC > KA (C) hA = hC; KB = KC (D) hA < hC; KB > KC
4. A sphere is rolling without slipping on a fixed horizontal plane surface. In the figure, A is the point of contact, B
is the centre of the sphere and C is its topmost point. Then
C

              
(A) v C  v A  2(v B  v C ) (B) v C  v B  v B  v A (C) v C  v A  2 v B  v C (D) v C  v A  4 v B

Page 90 of 99
Physics
ASSERTION - REASON
1. Statement–I : Two cylinders, one hollow (metal) and the other solid (wood) with the same mass and identical
dimensions are simultaneously allowed to roll without slipping down an inclined plane from the same height.
The hollow cylinder will reach the bottom of the inclined plane first.
and
Statement–2 : By the principle of conservation of energy, the total kinetic energies of both the cylinders are
identical when they reach the bottom of the incline.
(A) Statement-1 is True, Statement-2 is True; Statement-2 is a correct explanation for
Statement-1
(B) Statement-1 is True, Statement-2 is True; Statement-2 is NOT a correct explanation for
Statement-1
(C) Statement-1 is True, Statement-2 is False
(D) Statement-1 is False, Statement-2 is True
COMPREHENSION BASED QUESTIONS
Comprehension #1
Two discs A and B are mounted coaxially on a vertical axle. The discs have moments of inertia I and 2I
respectively about the common axis. Disc A is imparted an initial angular velocity 2  using the entire potential
energy of a spring compressed by a distance x1. Disc B is imparted an angular velocity  by a spring having
the same spring constant and compressed by a distance x2. Both the discs rotate in the clockwise direction.
x1
1. The ratio x is :-
2

1 1
(A) 2 (B) (C) 2 (D)
2 2
2. When disc B is brought in contact with disc A, they acquire a common angular velocity in time t. The average
frictional torque on one disc by the other during this period is :-
2I 9I 9I 3I
(A) (B) (C) (D)
3t 2t 4t 2t

3. The loss of kinetic energy during the above process is :-


I 2 I 2 I 2 I 2
(A) (B) (C) (D)
2 3 4 6

Page 91 of 99
Physics
Comprehension #2
A uniform thin cylindrical disk of mass M and radius R is attached to two identical massless springs of spring
constant k which are fixed to the wall as shown in the figure. The springs are attached to the axle of the disk
symmetrically on either side at a distance d from its centre. The axle is massless and both the springs and the
axle are in a horizontal plane. The unstretched length of each spring is L. The disk is initially at its equilibrium
position with its centre of mass (CM) at a distance L from the wall. The disk rolls without slipping with velocity
 
v 0  v 0 ˆi . The coefficient of friction is µ.

d
2d
v0
R

1. The net external force acting on the disk when its centre of mass is at displacement x with respect to its equilibrium
position is
2kx 4kx
(A) – kx (B) – 2kx (C)  (D) 
3 3
2. The centre of mass of the disk undergoes simple harmonic motion with angular frequency w equal to
k 2k 2k 4k
(A) (B) (C) (D)
M M 3M 3M
3. The maximum value of v0 for which the disk will roll without slipping is
M M 3M 5M
(A) µg (B) µg (C) µg (D) µg
k 2k k 2k

SUBJECTIVE QUESTIONS
1. A uniform circular disc has radius R and mass m. A particle, also of mass m, is fixed at a point A on the
edge of the disc as shown in the figure. The disc can rotate freely about a horizontal chord PQ that is at
R
a distance from the centre C of the disc. The line AC is perpendicular to PQ. Initially the disc is held
4
vertical with the point A at its highest position. It is then allowed to fall, so that it starts rotation about PQ.
Find the linear speed of the particle as it reaches its lower position.

C
R
4
P Q

Page 92 of 99
Physics
2. A man pushes a cylinder of mass m1 with the help of a plank of mass m2 as shown. There is no slipping
at any contact. The horizontal component of the force applied by the man is F, Find :
F1 m2

m1

(i) the acceleration of the plank and the centre of mass of the cylinder and
(ii) the magnitude and directions of frictional forces at contact points.
3. A rod AB of mass M and length L is lying on a horizontal frictionless surface. A particle of mass m travelling
along the surface hits the end A of the rod with a velocity v0 in a direction perpendicular to AB. The collision
is elastic. After the collision the particle comes to rest.
m
(i) Find the ratio
M
(ii) A point P on the rod is at rest immediately after collision. Find the distance AP.
L
(iii) Find the linear speed of the point P after a time after the collision.
3V0

4. Two heavy metallic plates are joined together at 90° to each other. A laminar
A
sheet of mass 30 kg is hinged at the line AB joining the two heavy metallic
plates. The hinges are frictionless. The moment of inertia of the laminar sheet Q
about an axis parallel to AB and passing through its centre of mass is 1.2
kg-m2. Two rubber obstacles P and Q are fixed, one on each metallic plate B

at a distance 0.5 m from the line AB. This distance is chosen, so that the
reaction due to the hinges on the laminar sheet is zero during the impact. Initially
the laminar sheet hits one of the obstacles with an angular velocity 1 rad/
s and turns back. If the impulse on the sheet due to each obstacle is 6
N-s.
(i) Find the location of the centre of mass of the laminar sheet from AB.
(ii) At what angular velocity does the laminar sheet come back after the first impact.
(iii) After how many impact, does the laminar sheet come to rest. y

5. Three particles A, B and C each of mass m, are connected to each other by three A x

massless rigid rods to form a rigid, equilateral triangular body of side l. This 
body is placed on a horizontal frictionless table (x-y plane) and is hinged to
it at the point A, so that it can move without friction about the vertical axis
through A (see figure). The body is set into rotational motion on the table F B C
about A with a constant angular velocity  .
(i) Find the magnitude of the horizontal force exerted by the hinge on the body.
(ii) At time T, when the side BC is parallel to the x-axis, a force F is applied on B along BC (as shown).
Obtain the x-component and the y-component of the force exerted by the hinge on the body, immediately
after time T.

Page 93 of 99
Physics
6. A rod of length L and mass M is hinged at point O. A small bullet of mass m hits the rod as shown in
the figure. The bullet gets embedded in the rod. Find angular velocity of the system just after impact.

mv

7. A solid cylinder rolls without slipping on an inclined plane inclined at an angle q. Find the linear acceleration
of the cylinder. Mass of the cylinder is M.
8. A uniform circular disc of mass 50 kg and radius 0.4 m is rotating with an angular velocity of
10 rad s–1 about its own axis, which is vertical. Two uniform circular rings, each of mass 6.25 kg and radius
0.2 m, are gently placed symmetrically on the disc in such a manner that they are touching each other along
the axis of the disc and are horizontal. Assume that the friction is large enough such that the rings are at
rest relative to the disc and the system rotates about the original axis. The new angular velocity
(in rad s–1) of the system is

Page 94 of 99
ANSWER KEY Physics

EXERCISE - 1 CLASS WORK


KINEMATICS
1. 20 s

2. (a) <w> = 2a / 3 = 4 rad/s; < β > = 3ab = 6 rad/s2;

(b) b = 2 3ab = 12 rad /s2.

3. t = 3 (4/a) tanα = 7 s

MOMENT OF INERTIA

100 2
4. 1.5 × 10–3 kg-m3, 4.0 × 10–3 kg-m3 5. (1  2 2  3 2  .........)  10  6 = 0.43 kg-m2
1000

M 2 14 mr 2
6. 7.
3 5

1  m 2  7
8.  9.
m  12  = 60
= 0.34 m I = (1/2) prbR4 = 2.8 g.m2]

 Aa 4 Ba 5 
2a7 m 2   
10. r/ 2 11. 12. 13. 2π  4 5 
21 6  

14. N = (aB – bA)k, where k is the unit vector of the z axis  = |aB –bA|/ A 2  B 2

1
15.  = |aB –bA|/ A 2  B 2 16. mg a sin θ
2

7 w mg
17. 18. P= cot q or P = cot q
13 2 2

21
19. 76 g, 42 g, . 20. 43 N
38

2g (m1  m 2 ) 60
(a) 
21. 7 = 8.4 rad/s
2
 (m1  m 2 )

2g (m1  m 2 ) 90
(b) (i)  (m  m  m / 3)  22 = 8.0 rad/s2,
1 2 3

 
(ii) (m 1 g – m1 a ) = 29 N ;(m 2 g + m2 a ) = 27.6 N
2 2

Page 95 of 99
Physics
3g 13mg 3 3 33
22. (a) (cw) (b) N = ­, F =  16  mg ® (c) 13
4L 16  

3g 3 Mg M  m  g
23. (a) 2 2  (cw) (b) g ¯ (c) ­ 24. I
2 4 M  m   R 2

25. 0.5 m/s 26. 0.9 2 dmg

ANGULAR MOMENTUM
27. 16 kg m 2/s 28. 0.5 kg-m 2/s, 75 J 29. 2.4 rad/s

4v 0 5v 0 v 2v 0
30. 106.7 rad/s 31. (a) (b) (c) vx = 0 , vy =
3 3  2 3

32. (a) wA = v2 / R = 2.0 m/s2, the vector wA is permanently directed to the centre of the wheel ;
(b) s = 8R = 4.0 m ]
6F
33.
5m

34. (a) vA = 2wt = 10.0 cm/s, vB = 2 wt = 7.1 cm/s, v0 = 0 ;


ω

(b) wA = 2w 1  ( t 2 /2R) 2 = 5.6 cm/s2,wB =w 1  (1  t 2 /R)2 = 2.5 cm/s2, w0 = w2t2 / R = 2.5 cm/s2

35. RA = 4r, RB = 2 2 r

36. (a) 300 mm from A (b) a = 4.00 rad/s2 (ccw) ; a = 1.800 m/s2 ®

 
37. m/s, m below the centre of the rod
6 6

2 7
38. 10 gh / 7 39. (a) tanq (b) mglsinq 40. 3.3 N
5 8

27
41. g(R  r ) 42. w = 3g (M + 3m) / (M + 9m + I/R2)
7

v0 2v 1. 7 v 2
43. (¬), 0 (®) 44. w= V = 0.3gR 45. wR/3
3 3 2R min 1.7

50 0
46. 3v/5 47 (a) (ccw) (b)
2r k g

Page 96 of 99
Physics
EXERCISE - 2 BASIC OBJECTIVE QUESTION
1. A 2. A 3. A 4. C 5. D 6. C 7. D

8. A 9. B 10. D 11. A 13. A 14. B 15. D

16. D 17. A 18. B 19. C 20. B 21. D 22. D

23. C 24. B 25. A 26. D 27. A 28. A 29. A

30. D 31. C 32. A 33. B 34. A 35. B,C,D,

36. D 37. B 38. (a) A (b) D 39. A 40. D 41. A

42. B 43. A 44. D 45. D 46. C 47. C 48. C

49. A 50. C 51. B 52. B 53. A 54. C 55. C

56. C 57. A 58. A 59. A 60. B 61. B 62. C

63. C 64. B 65. C 66. D 67. C 68. B 69. D

70. D 71. B 72. C

EXERCISE - 3 MAINS LEVEL EXERCISE


SECTION - A OBJECTIVE TYPE
1. B 2. B 3. C 4. C 5. B 6. B 7.` C
8. D 9. C 10. C 11. B 12. A 13. B 14. B
15. B 16. B 17. C 18. C

SECTION - B SUBJECTIVE TYPE


14Rg 12v
1. 133 2. 32M2 3. 4. (i) (ii) 3.5 m/s
3 7

 
1
5. 6.3 m/s 6. 7. (i) 1.63 N (ii) 1.22 m 8. (i) 6 ˆi N (ii) 0.6  ĵ  kˆ , 0.85 Nm
2 2

v 3mv 2 ML2
9. 12 rad/s2 10. (i) 0 (ii) anticlockwise (iii) E  11
1.
5a 5 L
2
 12d 2 
1
12. 3g 13. 1.2 second 14.    m down to the centre of mass

_________________________________________________________________________________________

Page 97 of 99
Physics
EXERCISE - 4 ADVANCED LEVEL EXERCISE
SECTION - A SINGLE CORRECT QUESTIONS
1. B 2. D 3. C 4. A 5. A 6. A 7. C
8. D 9. C 10. A 11. C 12. D 13. D 14. A
15. D 16. B 17. B 18. D 19. B 20. D 21. B
22. B 23. A 24. A 25. D 26. A

SECTION - B MULTIPLE CORRECT QUESTIONS


1. A,B,C,D 2. B,C 3. A,D 4. B,C,D 5. A,B,C
6. A,B,D 7. A,C,D 8. C 9. B,C,D 10. A
11. C 12. B,C 13. A,B,C 14. A,C 15. B,C
16. A,C,D 17. A,B,D

SECTION - C COMPREHENSION TYPE


Comprehension #1 : 1. C 2. B Comprehension #2 : 1. D
Comprehension #3: 1. B 2. A Comprehension #4 : 1. A 2. C
Comprehension #5 : 1. B 2. A Comprehension #6 : 1. C 2. A 3. B
Comprehension #7: 1. A 2. D 3. D Comprehension #8: 1. A 2. C 3. B 4. C
Comprehension #9 : 1.A,B,C 2.C 3.B,C,D 4.A,C,D

SECTION - D ASSERTION - REASON TYPE


1. B 2. D 3. B 4. D 5. D 6. A
7. D 8. D 9. A 10. D 11. A

SECTION - E MATRIX MATCH TYPE


1. (A) p,q,r (B) p,q,r (C) p,q (D) p,q,r 2. (A) q, (B) s, (C) q, (D) q
3. (A) q,t,u (B) q,t,u (C) q,t,u 4. (A) s, (B) r, (C) r
5. (A) q, (B) r, (C) p (D) r 6. (A) q, (B) p, (C) s (D) r

SECTION - F SUBJECTIVE QUESTIONS

 4 4gR
1. (i)   cos 1  7  (ii) (iii) 6
7

2 2
 7  2   3g   6g   13 2
(ii)    2   (iii)  7    ,  7 mg  m   (iv)
4 2 6g
2. (i) 1 2
mg
7  4g  7 7

3. 2 m/s, 1.5 m/s, 16.67 N

Page 98 of 99
Physics
1 2 2 1 2 2 v 0
4.  5 mr  m    0   5 mr   where  = =
2 2 2

2 2 r r

2
Vv Mv  mv  V  v    1 Mm   
5. (i) (ii) (iii)       (iv) (V  v ) 2 1  
 Mm 0 2 M m   0 

11
6. (i) kg (ii) 11 m/s 7. 2mw3r2
9

EXERCISE - 5 PREVIOUS YEAR QUESTIONS


SECTION - A MAINS LEVEL SINGLE CORRECT TYPE
1. C 2. A 3. B 4. D 5. A 6. D 7. B
8. B 9. C 10. D 11. D 12. C 13. B 14. C
15. C 16. D 17. C 18. A 19. C 20. D 21. D
22. D 23. D 24. B 25. D

SECTION - B ADVANCED LEVEL


SINGLE CORRECT QUESTION
1. A 2. A 3. C 4. C 5. B 6. C 7. D
8. A 9. B 10. C 11. A 12. A 13. A 14. B
15. D 16. A 17. A 18. D 19. B 20. A

MULTIPLE CORRECT QUESTIONS


1. A,B,C 2. C,D 3. A,B 4. B,C

ASSERTION - REASON
1. D

COMPREHENSION BASED QUESTIONS


Comprehension # 1 : 1. C, 2. A, 3. B Comprehension # 2 : 1. D, 2. D, 3. C

SUBJECTIVE QUESTIONS

4F 8F 3m 1 F m1F
1. 5gR 2. (i) acm = 3m  8m , aplank = 3m  8m (ii) 3m  8m  , 3m  8m 
1 2 1 2 1 2 1 2

1 2 v0
3. (i) (ii) L (iii) 4. (i) 0.1 m (ii) 1 rad/s (iii) Laminar sheet will never come to rest
4 3 2 2

F 3mv 2g sin 
5. (i) 3m2  (ii) Fx  , Fy = 3 m2  6. 3m  M L 7. 8. 8
4 3

Page 99 of 99

You might also like